Sie sind auf Seite 1von 304

CUARTO SIMULADOR DEL CURSO

ENARM CMN SIGLO XXI


INICIO DE SEGUNDA PARTE
Paciente masculino de 24 aos que recibe
usted en el servicio de urgencias por intento
de suicido, sin antecedentes personales
patolgicos de importancia. Quien a la
exploracin fsica se encuentra plido,
diafortico, taquicardico, somnoliento.

PREGUNTA 251
En presencia de prdida aguda de sangre, la
precarga cardiaca adecuada al principio de
mantiene por?:

RESPUESTA
a.- Desarrollo de taquicardia
b.- Efectos hormonales de la angiotensina
c.- Efectos hormonales de la renina
d.- Aumento de la resistencia vascular sistmica.
PREGUNTA 252
Se decide manejo quirrgico de urgencia para
exploracin de la lesin vascular de grandes vasos, y
por las condiciones generales se decide administrar
anestesia espinal alta; Posterior a este el paciente inicia
con piel seca, disminucin del pulso, motivo por el cual
se diagnstica en este momento choque neurognico. .
Este tipo de choque se caracteriza por:

RESPUESTA
a.- Piel hmeda y fra.
b.- Aumento del gasto cardiaco
c.- Descenso de la resistencia vascular perifrica
d.- Reduccin del volumen sanguneo.
Femenino de 47 aos que ingresa va urgencias
con una presin arterial de 70/50mmHg, nivel de
lactato srico de 30mg/100ml. Su gasto cardiaco
es de 1.9L/min y la presin venosa central de
2cmH2O.
PREGUNTA 253
Cul ser el diagnstico ms probable?
RESPUESTA
a.- Insuficiencia cardiaca congestiva
b.- Taponamiento cardiaco
c.- Choque hipovolmico
d.- Choque sptico

PREGUNTA 254
Posteriormente y con el paciente ya estabilizado
decide suministrar oxgeno 70%, toma
gasometra arterial y recaba: pH 7.48, Po2
55mmHg, PCo2 30mmHg. Estos resultados son
ms consistentes con el diagnstico de:
a.- Enfermedad pulmonar obstructiva crnica
b.- Dolor y ansiedad
c.- Sndrome de insuficiencia respiratoria del
adulto
d.- Atelectasia sbita


Se presenta en su consulta un paciente que refiere
sintomatologa que se orienta hacia un diagnstico de
Enfermedad por reflujo gastroesofgico. Posterior a
realizar estudios de laboratorio y gabinete usted
confirma dicho diagnstico.
PREGUNTA 255
La incidencia de epitelio metaplsico de Barrett que
evoluciona a adenocarcinoma es? :
RESPUESTA
a.- 1% por ao
b.- 3% por ao
c.- 5% por ao
d.- 10% por ao

PREGUNTA 256
El paciente al saber esta incidencia le pregunta
que si es candidato a manejo quirrgico y que
cul sera la mejor tcnica? Usted contestara:
RESPUESTA
a.- Miotoma de Heller
b.- Relajacin esofgica
c.- Funduplicatura de 360 grados
d.- Miotoma del esfnter esofgico inferior

Paciente masculino de 45 aos de edad quien
se encuentra hospitalizado en el servicio de
Ciruga General por el diagnstico de Ulcera
pptica. Actualmente el paciente ha tenido
una evolucin favorable, tolerando la va oral,
con uresis y excretas presentes. Afebril.
PREGUNTA 257
Cul de los siguientes compuestos secretan las
clulas principales gstricas?

RESPUESTA
a.- Somatostatina
b.- Gastrina
c.- Pepsingeno
d.- Histamina

PREGUNTA 258
En este mismo paciente usted corrobor la
presencia de Helicobacter pylori, motivo por el
cual dio tratamiento para la erradicacin del
mismo. Cul de las siguientes es la mejor prueba
para confirmar la erradicacin de la infeccin por
h pylori despus del tratamiento?
RESPUESTA
a.- Prueba serolgica
b.- Prueba de urea en aliento
c.- Estudio histolgico
d.- Prueba rpida de ureasa

Una mujer de 65 aos de edad con lcera
duodenal conocida, se trata con dieta y un
bloqueador H2. Ingresa al servicio de
urgencias con hemorragia gastrointestinal
superior de consideracin.
PREGUNTA 259
Despus de iniciar la reposicin sangunea, el
siguiente paso en su tratamiento sera?
RESPUESTA
a.- Iniciar bismuto y metronidazol
b.- Iniciar omeprazol
c.- Endoscopia y coagulacin del vaso sangrante
d.- Piloroduodenotoma y cierre del vaso
sangrante con sutura.

PREGUNTA 260
Usted considera que la paciente puede estar cursando
adems con un sndrome de Zollinger-Ellison, motivo
por el cual la prueba diagnstica ms exacta que usted
solicitara para corroborar su sospecha es:
RESPUESTA
a.- Gastrina srica en ayuno
b.- Tomografa computarizada
c.- Endoscopia
d.- Prueba de estimulacin con secretina

Paciente masculino de 76 aos que acude al
servicio de urgencias por iniciar su
padecimiento hace 48 horas por la presencia
de distensin abdominal importante, poca
tolerancia a la va oral, dificultad para
canalizar gases. Posterior a su valoracin
determina que el paciente cursa con una
obstruccin parcial de intestino delgado.
PREGUNTA 261
La sensibilidad de las radiografas de
abdomen para establecer el diagnstico de
obstruccin del intestino delgado es de?

RESPUESTA
a.- 30%-40%
b.- 50%-60%
c.- 70%-80%
d.- 90%-100%

PREGUNTA 262
Para este paciente el mejor estudio para
determinar el diagnstico de obstruccin parcial
del intestino delgado sera:

RESPUESTA
a.- Radiografas simples de abdomen
b.- Trnsito gastrointestinal
c.- Tomografa computarizada con contraste
d.- Resonancia magntica

Paciente femenino de 23 aos que ingresa va
urgencias por el diagnstico de abdomen
agudo. Posterior a su exploracin fsica,
estudios de laboratorio y gabinete determina
que la paciente cursa con un cuadro de
apendicitis, motivo por el cual decide
tratamiento quirrgico de urgencia.
PREGUNTA 263
Cules de las siguientes son localizaciones
comunes de la punta apendicular?

RESPUESTA
a.- Retrocecal, plvica, subcecal.
b.- Preileal, periclica derecha, infraovrica
c.- Retrocecal, subcecal, supracecal
d.- Periclica derecha, infraovrica, plvica

PREGUNTA 264
La paciente curso con un transoperatorio y
postoperatorio adecuado, tolerando la va oral a las 24
horas, canalizando gases. Cul es la duracin ptima
de los antibiticos posoperatorios luego del
procedimiento de apendicectoma?:
RESPUESTA
a.- 1-2 das para la apendicitis perforada
b.- 7-10 das para la apendicitis no perforada
c.- 7-10 das para la apendicitis perforada
d.- Hasta que el paciente est afebril y el recuento de
leucocitos sea normal

Masculino de 78 aos que ingresa va
urgencias por iniciar su padecimiento actual
posterior a varios eventos de vmitos hace 35
minutos con la presencia de sangrado de tubo
digestivo masivo. Antecedente de cirrosis
heptica e hipertensin portal secundaria a
ingesta de alcohol ya diagnosticada desde
hace 10 aos.
PREGUNTA 265
Cul de las siguientes medidas reduce el riesgo
de un sangrado de tubo digestivo alto inicial por
vrices en el paciente con hipertensin portal?
RESPUESTA
a.- Derivacin porto-cava intraheptica
transyugular
b.- Derivacin esplenorrenal
c.- Escleroterapia
d.- Bloqueo B

PREGUNTA 266
El sangrado del paciente es controlado de forma
eficaz posterior a su manejo inmediato, sin
embargo usted sabe que el riesgo de resangrado
por la hipertensin portal es alto. Qu
porcentaje del flujo sanguneo hacia el hgado
procede de la vena porta?

RESPUESTA
a.- 30%
b.- 50%
c.-75%
d.- 90%
Se trata de paciente masculino de 59 aos de
edad que ingresa a hospitalizacin a cargo del
servicio de Ciruga General por el diagnstico
de ictericia. Usted inicia protocolo de manejo
y lo primero que realiza es un ultrasonido de
hgado y vas biliares en donde le reportan
dilatacin del coldoco.
PREGUNTA 267
La siguiente prueba diagnstica para este
paciente que usted solicitara sera?:

RESPUESTA
a.- Tomografa computarizada
b.- Centellografia biliar
c.- Colangiopancreatografia retrograda
endoscpica
d.- Colangiografa transheptica percutnea

PREGUNTA 268
Finalmente usted decide realizar procedimiento
quirrgico mediante colecistectoma laparoscpica
con exploracin de va biliar. En el momento que
usted explora y abre la vescula biliar se encuentra
con la presencia de clculos biliares cafs, por lo cual
de acuerdo al color de los clculos usted supondra
que son debidos a:
RESPUESTA
a.- Son casi siempre de colesterol
b.- Se deben a una infeccin
c.- Se originan por un aumento de la bilirrubina
conjugada
d.- Se encuentran sobre todo en Sudamrica


Paciente masculino de 59 aos que es visto en
la consulta externa por referir iniciar su
padecimiento con la presencia de protusin
de una masa de aproximadamente 3x4cm,
posterior a realizar esfuerzo fsico intenso,
localizada a nivel lateral del borde lateral del
musculo recto abdominal del lado derecho.
Posterior a su exploracin fsica.
PREGUNTA 269
usted piensa que se trata de?

RESPUESTA
a.- Hernia umbilical gigante
b.- Hernia de lnea media abdominal
c.- Hernia inguino-escrotal
d.- Hernia de Spigel

PREGUNTA 270
Este tipo de hernia suele formarse a nivel de:

RESPUESTA
a.- Borde lateral del recto abdominal
b.- Lnea alba
c.- Pared medial del conducto inguinal
d.- Tringulo femoral

Se trata de paciente masculino de 39 aos de
edad quien se encuentra en el servicio de
Ciruga General con el diagnstico de
pancreatitis aguda biliar, quien se le instal
manejo mdico mediante ayuno, colocacin
de sonda nasogstrica. Sin embargo la
evolucin del paciente ha sido insidiosa
presentando con reaccin inflamatoria
sistmica caracterizada por fiebre y
leucocitosis.
PREGUNTA 271
Cul signo sera el ms relevante que lo obligara
a usted pensar en algn manejo quirrgico?

RESPUESTA
a.- Edad del paciente
b.- Tiempo de hospitalizacin
c.- Reaccin inflamatoria sistmica
d.- Pancreatitis aguda

PREGUNTA 272
Usted decide someter al paciente a realizacin de
aspiracin guiada por tomografa con crecimiento
de E. coli en el cultivo. Qu tratamiento sera el
ms adecuado?

RESPUESTA
a.- Antibitico ajustado al cultivo
b.- Laparotoma exploradora
c.- C.P.R.E.
d.- Colocacin de drenaje isobrico
PREGUNTA 273
Una mujer de 30 aos , por lo dems saludable, se
somete a apendicectomia con cierre primario de la
herida por apndice perforado. No se administraron
antibiticos. Si esta persona desarrollara un absceso
intraabdominal, Cul de los siguientes
microorganismos sera el causante ms probable?
RESPUESTA
a.- E. coli
b.- Bacteroides
c.- S. faecalis
d.- Serratia
PREGUNTA 274
En general, cul de las siguientes es la
infeccin ms frecuente en pacientes
quirrgicos hospitalizados?

RESPUESTA
a.- Gastrointestinal inferior
b.- Vas respiratorias inferiores
c.- Nasofarngea
d.- De la herida quirrgica.

Se trata de paciente femenino de 25 aos de
edad quien ingresa va urgencias por herida
por arma de fuego en miembro plvico
derecho. Posterior a la atencin primaria
brindada la paciente es estabilizada.
PREGUNTA 275
Cul de las siguientes situaciones llevara a realizar
una arteriografa o exploracin de una posible lesin
en la arteria femoral por la herida de arma de fuego?

RESPUESTA
a.- Proximidad del trayecto probable del proyectil al
paquete neurovascular
b.- Un pequeo hematoma en el sitio de entrada
c.- Presin sistlica con diferencia de 5% entre ambas
piernas
d.- Soplo sobre el sitio de la lesin

PREGUNTA 276
La paciente inicia con datos de hipotensin de manera
sbita, motivo por el cul usted decide comenzar con
manejo de lquidos. Qu porcentaje de sangre debe
perderse en pacientes sanos antes de que sobrevenga
la hipotensin?

a.- 10%-20%
b.- 20%-30%
c.- 30%-40%
d.- 40%-50%
PREGUNTA 277
Se trata de paciente masculino de 33 aos de edad que
acude al servicio de urgencias con diagnstico de
herida penetrante en el trax del lado derecho. Lo
primero que usted realizara sera:

RESPUESTA
a.- Toma de signos vitales.
b.- Toma urgente de tele de trax
c.- Control de la va rea
d.- Exploracin de la herida penetrante

PREGUNTA 278
En qu momento decidira someter al paciente a
realizacin de Toracotoma?
RESPUESTA
a.- Drenan ms de 1,000 ml de sangre por la sonda
torcica cuando se instala.
b.- La sonda torcica drena ms de 200ml por hora de
sangre durante 3 horas.
c.- Hay una fuga de aire que persiste ms de 48 horas
d.- Hay lesin pulmonar documentada en la TAC.
Despus de un accidente automovilstico, un
varn de 19 aos de edad ingresa a la sala de
urgencias con una lesin facial central
extensa. Su respiracin es laboriosa, tose y
expulsa sangre y moco en cada respiracin. Se
resiste y se muestra combativo.
PREGUNTA 279
El primer paso teraputico debe consistir en
mantener traccin cervical alineada y:

RESPUESTA
a.- Colocar una cnula endotraqueal por va nasal
b.- Instalar una cnula endototraqueal por va
oral
c.- Realizar cricotiroidotomia
d.- Practicar traqueostoma

PREGUNTA 281
El paciente, posterioemente inicia con presencia de
hemorragia profusa por una herida localizada por
debajo del ligamento inguinal derecho y est en
choque. El primer paso en el control local de la herida
debe ser?
RESPUESTA
a.- Aplicar compresin en el vaso sangrante con un
dedo enguantado
b.- Colocar un torniquete en el muslo derecho, por
arriba de la herida
c.- Usar pinzas y ligaduras para controlar la hemorragia
d.- Envolver la herida con vendaje compresivo

Se trata de paciente masculino de 65 aos de
edad que ingresa va urgencias con el
diagnstico de oclusin intestinal. Posterior a
manejo mdico dado por usted decide manejo
quirrgico de urgencia encontrando que la
causa de la oclusin es un tumor el cul
reseca, realizando posteriormente entero-
entero anastomosis para restablecer el tubo
digestivo
PREGUNTA 282
El tumor ms frecuente del intestino delgado
es?

RESPUESTA
a.- Carcinoma
b.- Adenoma
c.- Fibroma
d.- Hemangioma
PREGUNTA 283
El paciente es egresado del servicio por mejora.
El cncer ms comn del intestino delgado es?

RESPUESTA
a.- Tumor estromal gastrointestinal
b.- Tumor carcinoide
c.- Adenocarcinoma
d.- Linfoma
Ingresa paciente masculino de 29 aos de
edad con el diagnstico de enfermedad
diverticular a nivel del colon del lado derecho.
Se decide manejo quirrgico realizando
diverticulectomia sin eventualidades
postquirrgicas. El paciente es egresado del
servicio por mejora.
PREGUNTA 284
Los divertculos colnicos del lado derecho:

RESPUESTA
a.- Se presentan en paciente geritricos
b.- Son ms frecuentes en personas de origen
asitico
c.- Casi siempre pueden tratarse con
diverticulectomia simples
d.- Por lo general se presentan con hemorragia
gastrointestinal baja
PREGUNTA 285
La bacteria ms frecuente en el colon es:

RESPUESTA
a.- Bacteroides
b.- C. difficile
c.- E. coli
d.- Salmonella

Se atiende mujer de 80 aos con dolor
abdominal y constipacin. En la exploracin
fsica se encuentra afebril, con taquicardia
ligera. El abdomen esta distendido y
timpnico, pero no existen signos
peritoneales. Las radiografas abdominales
sugieren la presencia de vlvulo sigmoideo.
PREGUNTA 286
El primer paso teraputico consiste en:

RESPUESTA
a.- Administracin de laxantes y enemas de
limpieza
b.- Enema con bario
c.- Sigmoidoscopia rgida
d.- Reseccin sigmoidea

PREGUNTA 287
El sitio ms frecuente de vlvulo es:

RESPUESTA
a.- Ciego
b.- Yeyuno proximal
c.- Colon sigmoides
d.- Estmago
PREGUNTA 288
Se trata de paciente masculino de 65 aos de
edad que ingresa va consulta externa con el
diagnostico de Cancer prosttico, programado
para reseccin prosttica. En qu zona de la
prstata es ms frecuente el cncer prosttico?

RESPUESTA
a.- Central
b.- Ureteral
c.- Transicin
d.- Perifrica
PREGUNTA 289
Usted decide solicitar laboratorios pre quirrgicos
y le interesa medir la funcin renal. La mejor
forma de medir esta funcin en el paciente de
forma preopertoria es:

RESPUESTA
a.- Urea, nitrgeno y creatinina en suero
b.- Depuracin de creatinina
c.- Ecografa renal
d.- TAC

Masculino de 30 aos que acude al servicio de
urgencias con sintomatologa de infeccin de
vas urinarias. Posterior a interrogatorio,
historia clnica corroboran el diagnstico e
inician medicamentos para el manejo de dicha
patologa.
PREGUNTA 290
El tratamiento debe de durar:

RESPUESTA
a.- 3 das
b.- 7 das
c.- 10 das
d.- 21 das

PREGUNTA 291
Al realizar la exploracin fsica usted descubre una
masa testicular del lado derecho de aproximadamente
2x3cm, dolorosa. El tratamiento apropiado ser:

RESPUESTA
a.- Biopsia transescrotal con aguja
b.- Biopsia transescrotal abierta
c.- Biopsia inguinal abierta
d.- Orquiectoma radical

CASE REPORT

A 56-year-old female presents for a health
maintenance examination. She has a history of a
total hysterectomy for benign disease 4 years ago.
You are able to document that the hysterectomy
pathology was benign and that she has had normal
Papanicolaou (Pap) tests for 10 years. The patient
asks about regular Pap smears.
QUESTION 292
Which one of the following would be the most
appropriate recommendation?

ANSWER
a) Routine Pap smears should be continued until age
70
b) A Pap smear should be done every 3 years
c) A Pap smear should be done yearly for 3 years and
only if indicated thereafter
d) A Pap smear is not indicated

QUESTION

A 31-year-old African-American female presents with
the chief complaint of bilateral galactorrhea of 3
months duration. She also has not menstruated for
1 year despite changing birth control pills several
times. A review of systems is otherwise
noncontributory. Except for a milky discharge with
stimulation of the breasts, her examination is within
normal limits. Serum prolactin on two occasions is
>200 :g/L (N 020).
QUESTION 293
Which one of the following would be most
appropriate at this point?

ANSWER
a) Stop her oral contraceptive and repeat the serum
prolactin level in 1 month
b) Start the patient on risperidone
c) Order bilateral mammography
d) Order a brain MRI with enhancement and
emphasis on the pituitary fossa
CASE REPORT
An 84-year-old African-American female is brought to
your office by her daughter, who is concerned that the
mother has memory problems and is neglecting to pay
her monthly bills. The mother also is forgetting
appointments and asks the same questions repeatedly.
This problem has been steadily worsening over the last
12 years. The patient has very little insight into her
problems, scores 24 out of a possible 30 points on the
Mini-Mental State Examination, and has difficulty with
short-term recall and visuospatial tasks. Her physical
examination and a thorough laboratory workup are
normal. A CT scan of the brain mreveals diffuse
atrophy.

QUESTION 294
Which one of the following is the most likely etiology
for this patients memory problem?

ANSWER
a) Alzheimers disease
b) Dementia resulting from depression
c) Normal aging
d) Lewy body dementia


CASE REPORT
A 23-year-old female is at 8 weeks gestation with her
first pregnancy. She is planning to travel to Third
World countries soon for job-related reasons.
QUESTION 295
Which one of the following is contraindicated in this
patient?
ANSWER
a) Hepatitis B vaccine
b) Rabies vaccine
c) Varicella vaccine
d) Meningococcal vaccine

QUESTION 296
In the evaluation of foot ulcerations, a neuropathic
etiology is suggested by which one of the following?

ANSWER
a) Abnormal monofilament testing
b) Absence of toe hair
c) Erectile dysfunction
d) Distal foot pain when supine

QUESTION 297
A 72-year-old white male has new-onset hypertension
with a current blood pressure of 190/110mm Hg.
Which one of the following agents can be used as
part of a test for diagnosing renovascular
hypertension, but would also increase the risk for
azotemia if used for treatment?

ANSWER
a) Furosemide
b) Metoprolol
c) Captopril
d) Amlodipine
QUESTION 298
Which one of the following treatments for childhood
nocturnal enuresis has both the highest cure
rates and the lowest relapse rates?

ANSWER
a) Positive reinforcement
b) Imipramine
c) Responsibility training
d) Bed-wetting alarms
CASE REPORT

You see a 30-year-old male who has just fallen on an
outstretched hand. He complains of wrist pain and
edema. Examination reveals tenderness over the
anatomic snuffbox and over the scaphoid tubercle at
the proximal wrist crease with the hand in extension.
Radiographs of the wrist are negative.
QUESTION 299
Which one of the following would be the most
appropriate action at this point?

ANSWER
a) Order high-spatial-resolution ultrasonography of
the wrist
b) Order a bone scan for the next day
c) Immobilize in a thumb spica splint for 12 weeks
and then order repeat radiographs
d) Immobilize in a cast for 68 weeks


CASE REPORT
A 15-month-old male is brought to the emergency
department following a generalized tonic-clonic
seizure at home. The parents report that the
seizure lasted 5 minutes, with confusion for the
next 15 minutes. This is the childs first seizure.
There is no family history of seizures. His medical
history is normal except for a 1-day history of a
URI. While initially lethargic in the emergency
department, the child is now awake and playful,
with a temperature of 39.5 C (103.2 F) and a
normal examination. Appropriate diagnostic tests
are performed, including a blood glucose level,
which is 96 mg/dL.
QUESTION 300
Which one of the following would be most appropriate
to administer at this point?

ANSWER
a) Ceftriaxone intravenously
b) Phenobarbital orally
c) Carbamazepine orally
d) Acetaminophen orally

CASE REPORT
A 77-year-old white male complains of urinary
incontinence of more than one years duration. The
incontinence occurs with sudden urgency. No
association with coughing or positional change has
been noted. There is no history of fever or dysuria.
One year ago he underwent transurethral resection
of the prostate (TURP) for benign prostatic
hypertrophy and says his urinary stream has
improved. Rectal examination reveals a smoothly
enlarged prostate without nodularity, and normal
sphincter tone. No residual urine is found with post-
void catheterization.
QUESTION 301
Which one of the following is the most likely cause of
this patients incontinence?

ANSWER
a) Overflow
b) Urinary tract infection
c) Fecal impaction
d) Detrusor instability

Mujer de 66 aos de edad, alrgica a la penicilina.
Ingresa para ciruga electiva por antecedente de
Enfermedad Divertcular. Se le practic
sigmoidectoma laparoscpica. Como profilaxis
antibitica se utiliz Clindamicina y Gentamicina.
Durante el postoperatorio presenta un cuadro de
dolor abdominal, febrcula y diarrea. En los
anlisis aparece leucocitosis y desviacin a la
izquierda. Se practica tomografa computarizada,
con resultado normal. Se decide reintervenir ante
la progresin del cuadro, sin evidencia
alteraciones en anastomosis. La paciente cae en
Sepsis y fallece. Reporte de Anatoma Patolgica
con datos de Colitis Pseudomembranosa.

PREGUNTA 302
El agente etiolgico de la Colitis Pseudomembranosa
es:

RESPUESTA
a.- C. difficile.
b.- E. coli.
c.- Enteroco fecalis.
d.- Enterobacter.

PREGUNTA 303
Utilidad de la biopsia para el diagnstico?

RESPUESTA
a.- > sensibilidad que TAC
b.- > sensibilidad que TAC
c.- = sensibilidad que IRM
d.- Ninguna
PREGUNTA 304
Toxina relacionada con la severidad de la Colitis
Pseudomembranosa ?

RESPUESTA
a.- Toxina A
b.- Toxina B
c.- Toxina Binaria
d.- A y B
PREGUNTA 305
Tratamiento de eleccin en cuadro agudo por Colitis
Pseudomembranosa

RESPUESTA
a.- Vancomicina 125mg IV cada 6 hr
b.- Metronidazol 500mg cada 6 hr VO
c.- Vancomicina 100mg IV cada 12hr +
Metronidazol 500mg cada 8 horas VO
d.- TMP/SMX 160/80mg cada 12 hr IV

PREGUNTA 306
El paciente contina con deterioro clnico con
aumento del dolor abdominal, fiebre, no presenta
evacuaciones, se decide cambiar a:

RESPUESTA
a.- Vancomicina 125mg VO cada 6 hr
b.- Metronidazol 500mg cada 6 hr VO
c.- Vancomicina 100mg IV cada 12hr + Metronidazol
500mg cada 8 horas VO
d.- TMP/SMX 160/80mg cada 12 hr IV
Femenino de 66 aos con antecedente de DM y
HAS mal controladas. Acude al servicio de
urgencias por cuadro de dolor abdominal de 4
das de evolucin en fosa iliaca izquierda,
consult a facultativo hace 3 das quien indico
Butilhioscina y Ciprofloxacino por una probable
GEPI, sin embargo, no presenta mejora con
agudizacin del dolor. EF abdomen globoso
depresible con dolor en FII y Flanco izquierdo,
rebote dudoso, TA 135/89mmHg, FC 91, FR 36, T
38.2C.
PREGUNTA 307
El probable diagnstico es:

RESPUESTA
A: Apendicitis aguda
B: Cncer de colon abscedado
C: Diverticultis aguda
D: Fistula Recto-Vaginal

PREGUNTA 308
Usted indica como primer estudio de
extensin:

RESPUESTA
A: Placa simple de abdomen
B: Ultrasonido abdominal
C: TAC de abdomen contrastada
D: Colon por enema

PREGUNTA 309
Reporte de Absceso Plvico septado con un
aproximado de 8cc en su interior, el manejo es:

RESPUESTA
a) Mdico con Antibiticos Intrahospitalario
b) Quirrgico con Procedimiento de 1 solo tiempo
c) Quirrgico con Procedimiento de Hartmann
d) Mdico con Antibiticos Ambulatorio

PREGUNTA 310
Una vez resuelto el cuadro usted indica:

RESPUESTA
a) Dieta con abundante fibra y lquidos
b) Dieta sin irritantes y condimentos
c) Interconsulta al Cirujano Colorrectal para
manejo
d) Interconsulta a Cirujano Colorrectal para su
restitucin

Femenino de 62 aos antecedente de HAS en
control con Captorpil, FA en control con
Verapamil y AAS desconoce dsis. Inicia su PA
hace 2 aos con sangrado rojo fresco durante y
posterior a las evacuaciones as como dolor
durante la evacuacin. Tratada con Proctoacid
cada 12 horas durante 2 semanas. Exploracin
Fsica ano limpio cerrado con Hemorroide
externa tipo pliegue anterior no complicado, TR
tono y calibre conservado, Anoscopia con
paquetes hemorroidales Grado II con laceracin
de mucosa. RSCR sin alteraciones hasta 15cm

PREGUNTA 311
Cual es el diagnstico presuntivo?

RESPUESTA
a) Hemorroides
b) Enfermedad Hemorroidal con sangrado
c) Enfermedad Hemorroidal Mixta
d) Hemorroides Internas Complicadas

PREGUNTA 312
Cual es el tratamiento mas adecuado para el
caso?

RESPUESTA
a) Mdico con Lidocaina con Hidrocosrtisona por 7
das
b) Mdico con Medidas Higienico- Dietticas
c) Mdico con Ligadura con Banda Elastica
d) Quirrgico

Masculino de 51 aos con antecedentes de DM
contralada. Referida de Dermatologa por una
Pstula perianal la cual presenta periodos de
inflamacin, dolor y salida de secrecin
purulenta desde hace 5 meses. EF Consiente,
tranquilo, adecuado estado general, Signos
vitales con TA 135/88mmHgm FC 72, FR 20, T
38C. Ano limpio cerrado con orificio secundario
lateral izquierdo a 1.5cm del margen anal con
salida de material purulento. TR tono y calibre
conservado no masas ni fluctuaciones se palpa
depresin anterolateral izquierda discretamente
dolorosa a la palpacin.

PREGUNTA 313
Cual es el diagnstico mas probable?

RESPUESTA
a) Absceso Perianal
b) Fstula Ano-recto-perineal
c) Fstula Ano-recto-perineal abscedada
d) Hemorroide abscedada

PREGUNTA 314
Cual es el tratamiento mas adecuado?

RESPUESTA
a) Quirrgico con Hemorroidectoma
b) Quirrgico con Fistulectoma
c) Quirrgico con Fisurectoma
d) Mdico con Antibitico y analgsico

Femenino de 31 aos acude por dolor anal de
2 semanas de evolucin. Antecedente de
Fisura Anal y Enfermedad Hemorroidal Mixta
tratada con Daflon 200mg con mnima
mejora. La paciente refiere dolor intenso al
evacuar y sensacin de desgarro durante la
evacuacin as como masa perianal. Sangrado
ocasional (2 semana) manchando nicamente
el papel sanitario.
PREGUNTA 315
Cual es el tratamiento de eleccin mas adecuado?

RESPUESTA
a) Ligadura Hemorroidal de urgencia
b) Hidrosmina 200mg cada 8 horas VO por 10 das
c) Nitroglicerina crema 2% cada 12 horas por 6 semanas
d) Analgsicos, sediluvios y ciruga posterior a la crisis

Mujer de 54 aos que, desde hace 6 horas,
presenta vmitos intensos y dolor abdominal.
En la exploracin clnica se aprecia tumoracin
umbilical dolorosa y en la Rx simple,
dilatacin de asas de I. delgado.

PREGUNTA 316
Entre los siguientes, diagnstico ms probable es:

RESPUESTA
a) 1. Tumor de intestino delgado.
b) 2. Invaginacin intestinal.
c) 3. Hernia estrangulada.
d) 4. Estenosis pilrica.

CASE REPORT

In early February, you receive a call from your office
nurse. Her 5-month-old daughter has been ill for
several days. What started as a mild upper
respiratory infection has progressed and she now has
profuse rhinorrhea, a temperature of 100.2 F (37.9
C), and audible wheezing. In spite of an almost
nonstop cough, she does not appear acutely ill.
QUESTION 317
The organism responsible for this childs illness is
most likely to be?

ANSWER
a) Bordetella pertussis
b) respiratory syncytial virus
c) group B Streptococcus
d) parainfluenza virus 3


QUESTION 318
Of the following, which is the most frequent cause of
seizures in the elderly?

ANSWER
a) Stroke
b) Head trauma
c) Dementia
d) Alcohol withdrawal

CASE REPORT

A 4-week-old full-term male is brought to your office by
his parents. They report that their child started
vomiting just after his 1-week visit. The parents are
concerned because they think the vomiting is
worsening, occurring after every feeding, and
shooting across the room. You note that the baby is
afebrile, but has not gained any weight since birth.
QUESTION 319
A 32-year-old gravida 3 para 2 is in labor at term following
an uncomplicated prenatal course. As you deliver the
fetal head it retracts against the perineum. Downward
traction fails to free the anterior shoulder. The most
appropriate course of action would be to?

ANSWER
a) have an assistant apply fundal pressure
b) place the mothers thighs on her abdomen
c) apply increasingly strong downward traction to the fetal
head
d) deliberately fracture the clavicle of the fetus

CASE REPORT

A 14-year-old male is brought to your office by his
mother to establish care. The patient has been
diagnosed with asthma, but has not been on any
medications for the past year. When questioned, he
reports that his asthmatic symptoms occur daily and
more than one night per week. On examination, he is
found to have a peak expiratory flow of 75%.
QUESTION 320
Based on these findings, the most accurate
classification of this patients asthma is?

ANSWER
a) mild persistent
b) moderate persistent
c) severe persistent
d) mild intermittent
QUESTION 321
Which one of the following Papanicolaou (Pap) test
results is most likely to indicate a cancerous lesion?

ANSWER
a) High-grade squamous intraepithelial lesion (HSIL)
b) Atypical glandular cells not otherwise specified
(AGC-NOS)
c) Low-grade squamous intraepithelial lesion (LSIL)
d) Atypical squamous cells cannot exclude high-grade
intraepithelial lesion (ASC-H)

CASE REPORT

A 72-year-old female has stable but moderately
severe COPD requiring 2 L of continuous oxygen.
She plans to attend the college graduation of her
first grandchild, and wants to fly to avoid a 12-hour
car ride. Her PaO on room air is 55 mm Hg.

QUESTION 322
According to the Federal Air Regulations, she MUST?

ANSWER
a) find a form of transportation other than air travel
b) bring a medical certificate from you certifying that
she is cleared to fly without oxygen
c) arrange through the airline for oxygen to be
available on board and in the airports
d) undergo preflight testing, including pulmonary
function testing and high-altitude simulation
testing, before she can be cleared to fly


QUESTION 323
Which one of the following criteria is most likely to signify
that an individual has bulimia nervosa rather than
anorexia nervosa?

ANSWER
a) The absence of at least three consecutive menstrual
cycles in a postmenarchal nonpregnant female
b) Recurrent episodes of binge eating
c) Intense fear of weight gain or becoming fat, even
though underweight
d) Denial of the seriousness of the current low body
weight

QUESTION 324
Painful ingrown toenails that display granulation tissue
and lateral nail fold hypertrophy are best
treated by?

ANSWER
a) removal of the entire nail
b) cotton-wick elevation of the affected nail corner
c) antibiotic therapy
d) excision of the lateral nail plate combined with
lateral matricectomy

CASE REPORT

A 36-year-old female makes an appointment because
her husband of 12 years was just diagnosed with
hepatitis C when he tried to become a blood donor
for the first time. He recalls multiple blood
transfusions following a motorcycle crash in 1988.
His wife denies past liver disease, blood transfusions,
and intravenous drug use. She has had no other
sexual partners. The couple has three children.
QUESTION 325
Which one of the following is the best advice about
testing the wife and their three children?

ANSWER
a) No testing is required if her husband has normal
liver enzyme levels
b) No testing is required because tests have low
sensitivity
c) All family members should be tested because of
possible household fecal-oral spread
d) She should be offered testing because sexual
transmission is possible


QUESTION 326
You are evaluating a 5-month-old with fever, tachypnea,
and mild respiratory distress in the emergency
department. You hear mild basilar rales. The child
does not appear toxic. Which one of the following
tests would be the most appropriate as an initial
study?

ANSWER
a) A C-reactive protein level
b) A chest radiograph
c) A CBC
d) Oxygen saturation by pulse oximetry
QUESTION 327
The elderly patient may process medications differently
than a younger adult. Which one of the following is
LEAST affected by aging?

ANSWER
a) Absorption
b) Distribution
c) Compliance
d) Metabolism
QUESTION 328
A 72-year-old Asian female is found to have asymptomatic
gallstones on abdominal ultrasonography performed to
evaluate an abdominal aortic aneurysm. Which one of
the following would be the most appropriate
management for the gallstones?

ANSWER
a) Lithotripsy
b) Laparoscopic cholecystectomy
c) Open cholecystectomy
d) Observation

A 34-year-old female presents to you for
preconception counseling regarding the
management of her chronic hypertension. Her
blood pressure has been well controlled on
benazepril, 20 mg/day, without any side effects.
The patients blood pressure was 145/95 mm Hg
prior to beginning benazepril. She has been
pregnant once before, and her physician switched
her to methyldopa during that pregnancy, but she
suffered from drowsiness and a dry mouth during
much of that time. The pregnancy and delivery
were otherwise uncomplicated. She has no
history of diabetes mellitus, renal insufficiency, or
asthma. She is a nonsmoker.
QUESTION 329
Which one of the following would you do when she
becomes pregnant?

ANSWER
a) Switch to atenolol until after delivery
b) Switch to long-acting nifedipine until after delivery
c) Continue the benazepril through the pregnancy and
delivery
d) Discontinue the benazepril and monitor closely
throughout the pregnancy for signs of
preeclampsia or fetal growth restriction
QUESTION 330
Which one of the following is the preferred treatment
for patients with obsessive-compulsive disorder?

ANSWER
a) Valproic acid
b) Alprazolam
c) Fluoxetine
d) Lithium carbonate

QUESTION 331
When repairing a perineal laceration after a vaginal
delivery, which one of the following suture materials
decreases both wound dehiscence and postpartum
perineal pain?

ANSWER
a) Polyglactin 910 (Vicryl)
b) Silk
c) Plain catgut
d) Polypropylene

QUESTION 332
A 40-year-old female is scheduled for a
cholecystectomy and you wish to estimate her risk
for postoperative bleeding. Which one of the
following provides the most sensitive method for
identifying her risk?

ANSWER
a) Prothrombin time (PT)
b) Activated partial thromboplastin time (aPTT)
c) Bleeding history
d) Bleeding time

QUESTION 333
A mother is nearing the end of maternity leave. She
asks for your advice regarding breastfeeding her
infant after she returns to work. Which one of the
following would be accurate advice?

ANSWER
a) Breast milk should be stored in glass bottles
b) Separation of breast milk indicates spoilage
c) Breast milk should be thawed in boiling water
d) Refreezing breast milk destroys some proteins
QUESTION 334
Compared to anesthesia using only parenteral opioids,
the use of epidural anesthesia in labor and delivery
increases the rate of which one of the following?

ANSWER
a) Low Apgar scores (<7)
b) Maternal low backache 3 months post delivery
c) Prolonged second stage of labor
d) Cesarean section

CASE REPORT

A 70-year-old white female presents with a pruritic rash
of her sacrum that has occurred intermittently over
the last 6 years. She reports that the area is always
very tender just before the blister-like lesions erupt.
She is otherwise in good health, and takes no
medications. Her past medical history is
unremarkable. You provide appropriate treatment
for the condition.
QUESTION 335
You should advise the patient to avoid which one of
the following during future outbreaks?

ANSWER
a) Sexual contact
b) Prolonged sitting
c) Sun exposure
d) Excessive intake of green, leafy vegetables

A 32-year-old female presents with bilateral
pretibial tender, mildly red nodules 24 cm in
diameter. A nodule that appeared earlier
resolved, leaving a bruised area. She had a
similar problem once when she was pregnant but
it resolved spontaneously. Her medications
include lovastatin (Mevacor) for hyperlipidemia
and a low-dose oral contraceptive prescribed 5
months earlier. Her past history and a review of
systems are otherwise unremarkable.
QUESTION 336
The most appropriate next step would be to

ANSWER
a) order a serum creatine phosphokinase level
b) discontinue her oral contraceptive
c) obtain a cervical culture for gonorrhea
d) discontinue lovastatin

QUESTION 337
A 23-year-old female was recently diagnosed with
bipolar disorder after experiencing her first episode
of acute severe mania. Which one of the following
would be appropriate initial maintenance therapy for
her?
ANSWER
a) No medication unless she has a second severe
episode
b) Valproate
c) Gabapentin
d) Lorazepam

CASE REPORT

A 23-year-old sexually active female presents to your
office with a 2-week history of vaginal discharge and
mild coital discomfort. On physical examination, you
note the presence of a mucopurulent vaginal
discharge and cervical friability. She is afebrile and
there are no other positive physical findings. No
trichomonads or yeast is seen on vaginal
preparations. Material for Chlamydia trachomatis
specific DNA testing is submitted and results will be
available in 2 days.

QUESTION 338
Which one of the following is true regarding appropriate
management?

ANSWER
a) No treatment is indicated until laboratory results are
known
b) metronidazole given now will clear her discharge
c) A 7-day course of doxycycline is superior to a 1-g dose
of azithromycin
d) The patient should be instructed to refrain from sexual
intercourse until 7 days after
initiating therapy


QUESTION 339
Compared to children with ADHD, adults with ADHD?

ANSWER
a) tend to be less impulsive
b) tend to be more hyperactive
c) are less likely to complain of inattention difficulties
d) are less likely to have corroboration of symptoms by
family members

QUESTION 340
A 33-year-old female requests combined oral
contraceptive pills (OCPs) for birth control. Which
one of the following would be a contraindication to
prescribing OCPs for this patient?

ANSWER
a) A family history of ovarian cancer
b) A history of controlled hypertension
c) A history of hepatitis C infection with no liver
disease
d) A history of thromboembolic disease

QUESTION 341
Routine blood tests frequently reveal elevated calcium
levels. When this elevation is associated with
elevated parathyroid hormone levels, which one of
the following is an indication for parathyroid surgery?

ANSWER
a) Concurrent hyperthyroidism
b) Increased bone density
c) Age >50
d) Kidney stones
QUESTION 342
A 21-year-old female had a pelvic examination and a
normal Papanicolaou (Pap) test 3 months ago. She is now
being treated for chlamydial cervicitis with azithromycin,
1 g in a single dose. When should she next have a test for
Chlamydia?
ANSWER
a) Never, unless she is symptomatic or has a suspected
exposure to Chlamydia at some point in the future.
b) 34 months
c) 12 weeks
d) 9 months (at her next routine pelvic examination)

Femenino de 21 aos de edad la cual acude a
urgencias secundaria a perdida del estado de
alerta, refiere compaantes que se encontraba en
clases cuando cayo de su propia altura sin causa
aparente, a la exploracin fsica se observa
mucosas orales deshidratadas, palides
generalizada, piel reseca, cabello quebradizo y de
cada fcil, se administra glucosa intravenosa y
mejora su estado, observa un estadio tanner no
correspondiente a la edad y al interrogatorio
refiere pirosis, disfagia y vomitos ocacionales con
trazas de sangre.

PREGUNTA 343
Cual es diagnostico ms probable de la paciente.

RESPUESTA
a.- Anorexia nervosa.
b.- Hipoglucemia.
c.- Desequilibrio hidroelectrolitico.
d.- Gastritis crnica.

Se trata de paciente de 24 aos de edad el cual ingresa
a urgencias por referir dolor en la regin cervical lo
cual le dificulta a la movilizacin de la cabeza, adems
refiere el acompaante que ha incrementado su
aislamiento social, como antecedentes de importancia
el paciente se encuentra diagnosticado con
esquizofrenia y actualmente recibe tratamiento con
Haloperidol 5 mg cada 8 horas, clonacepam 2 mg cada
24 horas y levomepromazina 25 mg en la noche,
agrega que hace 15 dias sali de internamiento
psiquitrico por brote psictico con una estancia de 3
semanas, no hay una red de apoyo adecuada. A la EF
observa rigidez en miembros superiores, reflejos
osteotendinoso incrementado, signo de rueda dentada
positivo, adems de sialorrea leve.

PREGUNTA 344
Cual es la conducta a seguir en el paciente.

RESPUESTA
a.- Reducir levomepromazina.
b.- Reducir haloperidol.
c.- Administrar biperiden.
d.- Administrar trihexifenidilo.

De trata de masculino de 79 aos de edad el cual se
encuentra diagnosticado con demencia tipo Alzheimer
desde hace 5 aos, actualmente permanece solo en
casa, con supervisin nocturna nicamente, se observa
con signos de desnutricin, con mal higiene
generalizada, a la exploracin fsica se observa con
ruidos cardiopulmonares adecuados, se observa con
moderada agitacin psicomotriz que se alterna con
periodos de somnolencia, no hay datos de focalizacin,
se realiza BH y EGO sin datos de procesos infecciosos,
se mantiene en observacin por la noche los sntomas
se agudizan con presentacin de ideas delirantes de
dao y alucinaciones visuales.

PREGUNTA 345
Cul es la conducta a seguir en este paciente.

RESPUESTA
a.- Realizar IRM de Craneo.
b.- Administrar haloperidol.
c.- Realizar hemocultivo.
d.- Cuantificacion de electrolitos sericos.

Ingresa paciente masculino de 32 aos de edad al
servicio de urgencias, se observa agitado, diafortico,
ansioso con sensacin de ahogo, refiere dificultad para
respirar con dolor torcico, y epigastralgia, refiere que
presenta temor a morir, refiere que su padre muri
hace 4 aos por infarto al miocardio as como su
hermano mayor el cual tena 36 aos de edad, a la
exploracin fsica se observa obesidad, TA 120/85, FC
102, FR 23, el paciente es agente de ventas y se
encuentra bajo presin continua, decide enviar
estudios de laboratorio y gabinete, observando
alteraciones en el ECG sin tener los resultados de
laboratorio.

PREGUNTA 346
Tomando en cuenta el cuadro clnico y la evidencia de
laboratorio y gabinete cual abordaje inicial.

RESPUESTA
a.- Indica oxigenoterapia y analgesia.
b.- Indica trombolisis, oxigenoterapia y analgesia.
c.- Indica anxiolitico, trombolisis, oxigenoterapia y
analgesia.
d.- Indica ansioltico y oxigenoterapia.
Femenino de 35 aos de edad la cual acude a
consulta por un resfriado comn, a la exploracin
se observa con la mirada fija, callada, disminucin
de la atencin y memoria a corto plazo
disminuida, a la exploracin fsica se observa
evasiva, hipoactiva, descuidada en su aspecto,
hace 3 aos se divorcio y actualmente solo tiene
trabajos temporales y ya ha perdido dos por
faltas injustificadas, refiere que ha comenzado a
ingerir bebidas alcohlicas hasta la embriaguez.

PREGUNTA 347
Usted considera que la paciente presenta un episodio
depresivo y considera iniciar tratamiento anterior a su
derivacin a psiquiatra, cual seria el mas adecuado:

RESPUESTA
a.- Imipramina.
b.- Fluoxetina.
c.- Duloxetina.
d.- Reboxetina.

Ingresa a urgencias femenino de 27 aos la cual se
encuentra bajo tratamiento por trastorno de
personalidad limite, ha sido tratada por mltiples
psiquiatras y esquemas teraputicos, al ingresar el
esposo de la paciente la encontr en la baera con
cortes superficiales en ambas muecas y falta de
respuesta a los estmulos sin embargo respira
superficialmente, en la sala de urgencias usted
identifica respiracin lenta y superficial sin percibir
olores especficos, observa ROTs disminuidos, con
respuesta ha estmulos dolorosos, su tratamiento
actual es clonacepam, clozapina y citalopram.

PREGUNTA 348
Considerando el manejo farmacolgico cual sera la
mejor intervencin para revertir el efecto de una dosis
excesiva ingerida en este intento suicida.

RESPUESTA
a.- La administracin de naloxona est indicada por
polifarmacia.
b.- Un agonista antagonista parcial competitivo como
flumacenil.
c.- Naltrexona es un frmaco que revierte la
dosificacin de benzodiacepinas.
d.- La paciente debe recibir biperiden para revertir los
efectos de clozapina.
Paciente femenino de 28 aos de edad la cual
se encuentra en pos-aborto de 5 dias, refiere
que ya no presenta sangrado, acude a solicitar
mtodo anticonceptico, refiere que no quiere
tener mas hijos por lo menos en 3 aos, tiene
2 hijos, anteriormente empleaba hormonales
orales y durante este mtodo se embarazo, no
cuenta con antecedentes de cervicovaginitis ni
EPI.

PREGUNTA 349
Cual de los siguientes mtodos de planificacin
familiar es el mas adecuado para este caso?

RESPUESTA
a.- Hormonal inyectado.
b.- Implante hormonal.
c.- Presenvativo.
d.- Dispositivo intrauterino.

PREGUNTA 350
La paciente acude ha consulta por cambios del
flujo menstrual, el cual refiere es obscuro y ha
presentado manchado intermenstrual, a la
exploracin se obseva moco cervical con sangre
moderada, cual es la conducta a seguir?

RESPUESTA
a.- Retirar el DIU.
b.- Indicar un AINES.
c.- Indicar antibitico.
d.- Indicar DOC.

Femenino de 31 aos de edad la acude a
consulta debido a que presenta amenorrea
secundaria, agrega que desea embarazarse,
como antecedentes cuenta con menarca a 21
aos con oligomenorrea, se observa con
clasificacin de Taner grado 3, refiere
presencia de leucorrea frecuente ha recibido
tratamiento, a la exploracin observa Talla
1.47 mts, Peso 48 kg. Adems cuello alado,
torax en escudo.

PREGUNTA: 351
Cul es la condicin ms frecuente de origen
gentico que presenta con los datos observados
en este caso.

RESPUESTA:
a.- Sindrome de Kallman.
b.- Sindrome de Turner.
c.- Sindrome de Klinifelter.
c.- Sindrome XXX.

Se trata de paciente femenino de 21 aos de
edad la cual acude por falta de menstruacin
durante los ltimos 6 meses, niega vida sexual
activa, sus antecedentes GO son menarca 11
aos, G: 0, a la exploracin se encuentra
caracteres sexuales normales para edad y
sexo, se observa sobrepeso por IMC, niega
flujo, refiere que en los 3 ltimos aos sus
periodos menstruales han sido irregulares,
niega cefalea u otros sntomas.

PREGUNTA 352
Cul de los siguientes diagnsticos es el ms
probable.

RESPUESTA
a.- Microadenoma PRL productor.
b.- Adenoma GH productor.
c.- Microadenoma ACTH productor.
d.- MEN.

Femenino de 41 aos de edad. Entre sus antecedentes mencion el
diagnstico de tuberculosis intestinal tres aos antes, siendo tratada
durante 8 meses con tratamiento especfico tetra-asociado, periodos
menstruales regulares y un parto por cesrea a los 32 aos. Se
hospitalizo por cuadro de varios aos de evolucin, con frecuentes
episodios de dolor abdominal difuso, distensin abdominal,
flatulencia, nauseas, vmitos y constipacin, mayor durante ciclos
menstruales. El cuadro que se intensifico durante el ltimo ao,
asocindose espordicamente deposiciones con estras de sangre
viva y perdida de 4 a 5 Kg de peso. Al ingreso, paciente en regulares
condiciones generales, presin arterial 120/80 mmHg, frecuencia
cardiaca 88 latidos por minuto, temperatura 36,6 C, el abdomen se
encontraba distendido, timpanizado a la percusin, chapoteo en
flanco y fosa iliaca derecha y borborigmos generalizados a la
auscultacin. Los laboratorios, rayos X de trax, abdomen y
ecografa abdominal de ingreso fueron normales, exceptuando la
sangre oculta en heces que fue (+). Las revisiones ginecolgica y
proctolgica no encontraron alteraciones.

PREGUNTA 353
Cul es el estadio en el que se encuentra la
paciente.

RESPUESTA
a.- Estadio I (Mnimo) - 1-5 puntos.
b.- Estadio II (Leve) - 6-15 puntos.
c.- Estadio III (Moderada) - 16-40 puntos.
d.- Estadio IV (Severa) - ms 40 puntos.

Una paciente de 47 aos que consult por dolor intenso, de
carcter cclico, de aproximadamente 1 ao de evolucin, localizado
en la FID. Los dolores comenzaban 3 o 4 das antes de cada
menstruacin y precisaba de grandes dosis de analgsicos para su
control. Como antecedentes de inters refera una salpingectoma y
una cua ovrica izquierdas por va laparoscpica, probablemente
por quistes endometrisicos, as como la reseccin por
histeroscopia de un endometrioma uterino 3 aos antes. En las
exploraciones complementarias, que incluyeron resonancia nuclear
magntica y trnsito gastrointestinal baritado, nicamente destac
un endometrioma en ovario izquierdo de 17 mm de dimetro, que
el servicio de ginecologa desestim para ciruga. Ante el
diagnstico de dolor crnico recurrente en FID de origen
desconocido y dados los antecedentes mdico-quirrgicos de la
paciente, se plante realizar una laparoscopia exploradora, en la
que se evidenci un quiste seroso ovrico izquierdo y un apndice
vermiforme normal, pero no se apreciaron tumores ni lesiones
intraabdominales, por lo que se opt por llevar a cabo la
apendicectoma profilctica.

PREGUNTA 354
Cuales son las evidencias diagnosticas de
laboratorio y gabinete mas frecuentes.

RESPUESTA
a.- Niveles plasmticos de CA 125.
b.- Glndulas endometriales.
c.- Estroma endometrial
d.- Macrfagos con inclusiones de hemosiderina

Mujer de 30 aos de edad present secrecin
maloliente vaginal, ardorosa o quemante durante
4 meses. La paciente fue diagnosticada por
primera vez con tricomoniasis vaginal. Haba
recibido cinco ciclos de metronidazol 500 mg por
va oral dos veces al da, la mayor duracin del
tratamiento fue de 10 das. Su ltimo curso de
metronidazol fue de 1 mes antes de su
presentacin. No haba antecedentes de otras
enfermedades de transmisin sexual. Ella estaba
tomando anticonceptivos orales. En el examen,
vulva con eritema, edema y excoriacin. El
examen con espculo, la pared vaginal era
eritematosa, edematosa y con secreciones
anormales.

PREGUNTA 355
Cul es al hallazgo mas frecuente en esta patologa.

RESPUESTA
a.- Desaparicin de lactobacilos y bacterias anaerobias.
b.- Clulas epiteliales con bordes mal definidos.
c.- Abundantes cocobacilos.
d.- Presentacin de cuatro o ms episodios en un ao.

Acude a consulta femenino de 33 aos de edad, refiere
que hace 10 das fue diagnosticada con enfermedad
inflamatoria, menciona que ha tomado el tratamiento
de forma irregular, regreso debido a que desde hace 24
horas inicio con dolor abdominal intenso, que se
incrementa cuando camina o hace algn movimiento,
fatiga, adinamia, a la exploracin fsica observa TA
110/70, FC 89, FR 23, Temperatura de 38.5 grados, mal
estado generalizado, abdomen con datos de irritacin
peritoneal, decide ingresarla y enva estudios de
laboratorio y gabinete debido a que considera que la
paciente presenta una complicacin:

PREGUNTA 356
Cul es la complicacin ms probable que
presenta la paciente:

RESPUESTA
a.- Peritonitis espontanea.
b.- Absceso tubo-ovrico.
c.- Salpingitis aguda.
d.- Ooforitis aguda.

Una mujer de 35 aos de edad, con alcohol
crnico y abuso de nicotina fue admitida con
un historial de dos das de evolucin de dolor
en fosa iliaca izquierda, as como las
temperaturas febriles de 38.8C y los
parmetros de inflamacin elevados. Una
historia previa de tuberculosis fue negada por
el paciente. En un examen por ultrasonido se
observo lesin de 5x6 cm masa qustica en el
rea del anexo izquierdo.

PREGUNTA 357
Cul es el agente etiolgico mas frecuente.

RESPUESTA
a.- Neisseria g.
b.- Chalmydia.
c.- Mycoplasma.
d.- Ureaplasma .

Mujer de 27 aos con antecedentes de
migraa y colecistectoma laparoscpica 2
aos antes, presenta alteraciones en la
cantidad y frecuencia del ciclo menstrual asi
como flujo intermenstrual recurrente, como
antecedentes la paciente le fue realizada
cesarea por placenta de implantacin baja
total hace 6 meses, hay antecedentes
familiares de cncer endometrial y quistes
ovricos.

PREGUNTA 358
Cual es la conducta a seguir para establer un
escrutinio diferencial.

RESPUESTA
a.- USG enfocndose en anexos.
b.- Realizar PIE.
c.- Realizar evaluacin de gonadotrofina.
d.- Biopsia endometrial.

Mujer de 49 aos con 3 hijos, viene a verlo debido en a
que presenta perodos abundantes, desde hace dos
aos ha notado que esto se ha incrementado de tal
forma que le ha generado incomodidades ya que
afirma que inunda la toalla los primeros das y duran
hasta 10 dias adems agrega cansancio crnico, astenia
y adinamia. A la exploracin fsica se observa palidez
de tegumentos, uas quebradizas, pelo frgil asi como
piel fra. A la exploracin GO se observa sangre obscura
en vagina con algunos coagulos, dolor a la palpacin y
movilizacin uterina, refiere que en su anterior DOC se
reportaron cambios inflamatorios recibiendo
tratamiento el cual no especifica. Cuenta con OTB,
finalmente refiere sangrado poscoital.

PREGUNTA 359
Cul es la conducta a seguir.

RESPUESTA
a.- Realizar USG.
b.- Biometria hemtica y ferritina serica.
c.- Histeroscopia.
d.- Biopsia endometrial.

Mujer de 29 aos de edad la cual refiere
sangrado poscoital la cual ha resultado levemente
dolorosa, como antecedentes presento menarca
a los 9 aos, IVSA a los 14 aos, ha tenido 4
parejas sexuales, es usuaria de mtodo hormonal
oral, agrega que ha presentado varios cuadros de
cervicovaginitis, la ultima DOC presento cambios
inflamatorios, tabaquismo y alcoholismo positivo.
A la exploracin ginecolgica se observa crvix
afresado, doloroso a la movilizacin.

PREGUNTA 360
Cul es la conducta a seguir.

RESPUESTA
a.- Repetir DOC.
b.- Realizar colposcopia.
c.- Enviar a clnica de displasias.
d.- Realizar citologa de base liquida.

Paciente de 45 aos, con antecedentes de hipertensin arterial
crnica en tratamiento con atenolol (50 mg/da) e
hidroclorotiazida 12,5 mg cada 24 hrs. Multpara, usuaria de
dispositivo intrauterino (T de cobre, por 7 aos). Consult por
hipermenorrea y dismenorrea, de intensidad progresiva y
resistente al tratamiento mdico (antiinflamatorios no
esteroidales y retiro del dispositivo intrauterino). Al examen
fsico presentaba un tero aumentado de tamao, con anexos
normales. Se solicit ultrasonografa transvaginal que confirm
la presencia de miomas mltiples, el mayor en el fondo uterino
de 4 cm de dimetro. El hemograma demostr anemia
ferropriva significativa (Hto. 26%). Dado el tamao de los
miomas y sntomas asociados (hipermenorrea, anemia severa
secundaria, dismenorrea y dispareunia intensa), se decidi
realizar un tratamiento quirrgico subtotal. Evolucion
satisfactoriamente, dndose de alta en buenas condiciones con
suplemento oral de hierro.

PREGUNTA 361
Cul es la conducta a seguir.

RESPUESTA
a.- Realizar citologa cervical 3 veces anualmente.
b.- Realizar citologa cervical normalmente.
c.- Se puede descontinuar la citologa cervical.
d.- Realizar citologa vaginal cada 2 o 3 aos.

Acude a consulta femenino de 23 aos de
edad que presenta falta de periodo menstrual
y desea saber si esta embarazada, la paciente
se dedica a su casa, vive y tiene relaciones con
su pareja desde hace 2 aos y quiere tener su
primer hijo, refiere que nunca ha sido regular,
a la exploracin fsica se observa con una talla
de 156 cm y peso de 74 kg, se observa acn en
cara y espalda con hirsutismo, se observa
obscurecimiento de cuello y axilas.

PREGUNTA 362
Cul es la causa ms probable de la amenorrea
en este caso.

RESPUESTA
a.- Hiperandrogenismo.
b.- Incremento de progesterona.
c.- Disminucin de estrgenos.
d.- Aumento de gonadotrofinas.

Se trata de paciente de 60 aos de edad la cual refiere
sangrado transvaginal, refiere que hace un par de
meses inicio con manchado transvaginal, el cual se
convirti en sangrado franco, la paciente cuenta con
antecedentes de inicio de vida sexual activa a la edad
de 14 aos, menarca a los 9 aos, gesta 5 para 3
abortos 2, adems de 3 parejas sexuales, es hipertensa
desde hace 10 aos con adecuado control, diabetes
mellitus desde hace 8 aos y dislipidemia. A la
exploracin fsica se observa paciente con leve palidez
de tegumentos, con ndice de masa corporal de 31,
signos vitales de TA 135/95, FC 83, FR 21,
normotermica, torax sin fenmenos agregados,
abdomen depresible, no doloroso al tacto vagina
bimanual utero de caractersticas normales, no
doloroso, con presencia de manchado del guante.

PREGUNTA 363
Cul es la conducta a seguir ms adecuada.

RESPUESTA
a.- Realizar USG.
b.- Indicar progestgenos.
c.- Realizar biopsia.
d.- Realizar legrado hemosttico.

Se trata de femenino de 25 aos de edad la cual
acude al servicio de urgencias con dolor en
ambas extremidades inferiores, clicos, ardor al
orinar, presencia de manchado transvaginal,
refiere que tiene tres das con esta
sintomatologa y decidi acudir porque se
incrementaron los sntomas, al interrogatorio
refiere contar con 6 semanas de embarazo por
FUM, actualmente bajo control prenatal con
mdico familiar, sus antecedentes GO son Gesta 2
para 1, Abortos 0, refiere que el embarazo previo
fue sin complicaciones al igual que el parto, niega
toxicomanas ni presencia de otra patologa.

PREGUNTA 364
Luego de ingresar a la paciente a observacin, se
realiza cuantificacin de gonadotrofinas observandoce
dentro del rango, disminuyendo la sintomatologa al
mantenerse en reposo, cual es la conducta a seguir:
RESPUESTA
a.- Mantener a la paciente en observacin debido a
que puede progresar la amenaza de aborto.
b.- Iniciar con medicacin antiespamodica y vigilancia
en casa con cita abierta.
c.- Dar indicacin de reposo absoluto y signos de
alarma con cita abierta.
d.- Indicar antibitico, indicar reposo, dar signos de
alarma y cita abierta.
Se trata de femenino de 19 aos de edad la cual se
encuentra en control prenatal, acude a su consulta de
control, refiriendo que desde hace una semana
presenta dolor en fosa iliaca derecha, sensacin de
pesantez y distencin abdominal, agrega que ha tenido
dificultad para la evacuacin, refiere que el da previo a
la consulta observa sangrado leve transvaginal, la
paciente es ingresada a urgencias por sospecha de
embarazo se realiza PIE resultando positiva, por lo que
es ingresada para observacin ya que el sangrado no
ha disminuido y continua el dolor abdominal.

PREGUNTA 365
Cul es la su conducta a seguir, considerando la
evolucin del paciente.

RESPUESTA
a.- Realizar Bh en busca de anemia y leucocitosis.
b.- Realizar USG transvaginal.
c.- Realizar cuantificacin de Gonadotrofinas.
d.- Cuantificacin de progesterona.

Femenino de 28 aos, gesta 3, para 1, cesarea 1,
acude a consulta a las 34 SDG. Presenta un USG
realizado a las 24 SDG, con peso fetal aproximado
de (650 g) apropiados para la edad gestacional.
Despus del examen, el paciente no recibi
atencin prenatal durante dos meses. Se realiz
nuevamente un USG donde observ restriccin
del crecimiento intrauterino 1,440 g, con bajo
nivel de liquido amnitico, temperatura 36,0
grados, la frecuencia del pulso 84/min, frecuencia
respiratoria 20/min y la presin arterial 120/80
mmHg.

PREGUNTA. 366
Cul de las siguientes observaciones es falsa respecto a
esta patologa.

RESPUESTA
a.- Constituye un elemento accesorio del feto
b.- Depende del desarrollo, crecimiento y maduracin.
c.- Su volumen vara fisiolgicamente.
d.- El apgar depende del volumen del liquido
amnitico.
Se trata de femenino la cual cuenta con 36 semanas de
gestacion la cual inicia con trabajo de parto desde hace
6 horas, actualmente presenta contracciones cada 3
minutos aproximadamente y con duracin de 50 a 60
segundos, a la exploracin se observa producto
transverso, a la exploracin vaginal se observa 10 % de
borramiento y 1-2 centmetros de dilatacin, no se
palpan estructuras oseas, sin embargo se observa
sangre fresca y en cantidad moderada, la paciente
cuenta con antecedentes gineco-obstetricos de cesarea
previa con periodo intergenesico de 10 meses.

PREGUNTA 367
Considerando el cuadro clnico, cual es la
complicacin ms probable que se presentara en
este caso.

RESPUESTA
a.- Desprendimiento de Placenta.
b.- Ruptura Uterina.
c.- Sufrimiento Fetal.
d.- Corioamnioitis
Se trata de femenino con 40 semanas de gestacin la
cual acude a consulta debido a que inicia trabajo de
parto, refiere que desde hace 2 horas inicia con dolor
en la regin abdominal baja tipo contracciones,
adems de presencia de sangrado leve pero continuo, a
la exploracin fsica usted palpa contracciones de 2 a 3
en 10 minutos con una duracin de 40 segundos
aproximadamente, al realizar tacto encuentra
borramiento del 40 % y dilatacin de 3 cm, sin
embargo presenta sangrado importante, las constantes
vitales de la madre se encuentra dentro de parmetros
normales, no as del producto el cual observa
frecuencia cardiaca aproximadamente de 160 a 190
lpm.
PREGUNTA 368
Cul de los siguientes factores de riesgo esta
altamente relacionado con desprendimiento de
placenta normoinserta.

RESPUESTA
a.- Periodo intergenesico.
b.- Aborto recurrente.
c.- Ruptura prematura de membranas
d.- Miomectomia.

Se trata de paciente femenino de 29 aos de
edad la cual acude a urgencias refiriendo contar
con 34 semanas de gestacin por FUM, no cuenta
con cuidados prenatales, agrega que este es su
tercer embarazo, previamente dos partos,
agregando que no presenta complicaciones en
sus embarazos o partos previos, a la exploracin
usted observa borramiento del 100% y dilatacin
de 8 cm, decide dar continuidad al trabajo de
parto, ya que considera que en caso de enviarla a
unidad de mayor complejidad colocara en riesgo
al binomio.

PREGUNTA 369
Considerando que la paciente cursa con un
trabajo de parto pretermino, cual es la
complicacin ms frecuente que el producto
puede presentar y por lo tanto tendr que enviar
a un segundo nivel posterior a asistir el parto.

RESPUESTA
a.- Hemorragia Periventricular.
b.- Enfermedad de Membrana Hialina.
c.- Enterocolitis Necrotizante.
d.- Sepsis Neonatal.
Se trata de paciente femenino de 26 aos de
edad la cual acude a consulta de control prenatal
actualmente contando con 21 semanas de
gestacin, refiere que ha presentado cefalea
ocasional, mareo y cansancio a la exploracin
fsica se observa edema de miembros inferiores,
la consulta anterior usted envi estudios de EGO,
QS y BH, los datos de relevancia fue Hematocrito
de 38, las constantes vitales son de FR 18, FC 89,
TA 160/90. MmHg. Con edema leve sin hematuria
ni proteinuria.

PREGUNTA 370
Cul es la conducta a seguir mas adecuada.

RESPUESTA
a.- Restriccin de lquidos y sal.
b.- Vigilancia estrecha de tencin arterial.
c.- Monitorizacin de vitalidad fetal.
d.- Iniciar con alfametildopa.

Paciente femenino de 37 aos de edad actualmente
cursando su tercer embarazo con 14 semanas por FUM,
acude a visita de control prenatal cuenta con 17 SDG por
USG, antecedentes personales patolgicos de pre-
eclampsia, adems obesidad previa al embarazo, cuenta
con antecedentes familiares de diabetes y obesidad por
parte de la madre e hipertensin por parte del padre, el
cual fallece por infarto al miocardio. A la exploracin fsica
se observa paciente gestante con obesidad grado I, sin
tratamiento previo, se enva EGO, QS y BH, donde se
observa triglicridos de 350, colesterol de 290, glucosa de
105 mg/dl, hemoglobina glucosilada de 7, tensin arterial
en dos ocaciones de 135/95 mmHg. Tratada con dieta
hiposodica, hipocalorica y restriccin de liquidos.

PREGUNTA 371
Cul es el factor ms importante que presenta la
paciente, para considerar Hipertension Arterial
Pre-existente.

RESPUESTA
a.- Trigliceridemia.
b.- Hipercolesterolemia.
c.- Obesidad.
d.- Cifras tensinales altas.

Se trata de femenino de 27 aos de edad la
cual cuenta con 22 semanas de embarazo,
acude a su tercera visita de control prenatal, al
revisar su expediente usted observa que ha
presentado por tercera ocasin de forma
independiente una tensin arterial de 140/90,
se observa edema de miembros inferiores,
proteinuria, cefalea constante.

PREGUNTA 372
Cual de las medidas no es adecuada.

RESPUESTA
a.- Alfametildopa.
b.- Hidralacina 50 mg cada 12 hrs.
c.- Dieta hiposodia y restriccin de liquidos.
d.- Captopril 25 mg cada 24 hrs.

Se trata de paciente de 19 aos de edad la cual acude
por primera vez a consulta por amenorrea secundaria
de 30 semanas por FUM, acude hasta ahora ya que
desconocan su embarazo en casa, la paciente refiere
mareo, dolor abdominal en barra, opresin de torax,
cefalea e irritabilidad, a la exploracin fsica se observa
escleras hiperemicas, leve rubicundez facial, rots
incrementados y edema de miembros inferiores,
proteinuria, se mide fondo uterino con 28 cm, con
disminucin de actividad fetal, los signos vitales de la
madre fueron FR 21, FC 96, TA 190/110 en 3 ocaciones.

PREGUNTA 373
Cul es la conducta inmediata a seguir con la
paciente.

RESPUESTA
a.- Identificar la viabilidad fetal.
b.- Busqueda de sndrome de HELLP.
c.- Sulfato de magnesio.
d.- Prepara a la paciente para Cesarea.

Se trata de femenino de 27 aos con 32 semanas
de gestacin que llega al servicio de urgencias por
presentar cefalea intensa, acufenos y fosfenos,
dolor abdominal en barra, adems dolor en
regin lumbosacra y sensacin de pesantes
plvica, a la exploracin fsica se observa fondo
uterino de 28 cm, obesidad grado I, ROTs
incrementado, edema de miembros inferiores
+++, al tacto observa dilatacin de 9 cm, con
presencia de moco con sangre, se observan
protenas en orina ++, nivel de conciencia
disminida, leve desorientacin temporo-espacial,
el familiar refiere que la encontr en el suelo con
somnolencia.

PREGUNTA 374
Cul es la conducta a seguir de forma inmediata.

RESPUESTA
a.- Indica Sulfato de Magnesio.
b.- Indica Hidralacina.
c.- Preparar para Cesarea.
d.- Preparar para Parto.

Se trata de femenino de 18 aos de edad con 32
semanas de gestacin por fecha de ltima
menstruacin, sin atencin prenatal, es llevada a
urgencias por presentar en casa perdida del
estado de alerta y con movimientos tipo crisis
convulsivas, a la exploracin se observa
somnolienta con respuesta a estmulos
dolorosos, ha sido estabilizada en sus constantes
vitales, los resultados de laboratorios son los
siguientes, plaquetas de 45,000, proteinuria +++,
DHL de 590, AST 239, ALT 432.

PREGUNTA 375
Tomando en cuenta la gravedad del caso, cual es
la mejor conducta a seguir inmediata.

RESPUESTA
a.- Realizar Cesrea.
b.- Sulfato de magnesio.
c.- Estabilizar plaquetas.
d.- Maduracin Pulmonar.

Paciente femenino de 39 aos de edad la cual acude a
consulta debido a que presenta amenorrea de 12
semanas de amenorrea, agrega que ha presentado leve
fatiga, anorexia con nausea matinal, usted enva
laboratorios de rutina presentando glucosa en sangre
de 250 mg/dl y hemoglobina glucosilada de 9.5,
adems recibe captopril 25 mg al dia, por hipertensin
arterial diagnosticada hace 3 aos. Al interrogatorio la
paciente refiere antecedentes de madre diabtica e
hipertensa, sus antecedentes GO refiere menarca a 13
aos, gesta 3, para 2, abortos 0, adems refiere que su
embarazo anterior hace 5 aos presento un
incremento de 14 kilogramos con producto que preso
3,750 grs. Por va cesrea a las 36 semanas de
gestacin.

PREGUNTA 376
Cul es la conducta no adecuada a seguir.

RESPUESTA
a.- Indicar hipoglucemiente.
b.- Suspender IECA.
c.- Indicar interrupcin de embarazo.
d.- Indicar alfametildopa.

Paciente femenino de 21 aos de edad, la cual
presenta 24 semanas de gestacin, acude a
control prenatal, refiere ardor al orinar, deseos
continuos de orinar, con urgencia, usted
identifico un incremento de peso hasta el
momento a partir de su embarazo de 10 kilos. Sus
antecedentes presentan, obesidad previa al
embarazo, padre diabtico actualmente con
insuficiencia renal en tratamiento con
hemodilisis, madre hipertensa bajo tratamiento.

PREGUNTA 377
Considerando el cuadro clnico cual es la
complicacion mas frecuente que el producto
puede presentar.

RESPUESTA
a.- Producto Macrosomico.
b.- Diestres respiratorio.
c.- Hipoglucemia.
d.- Parto pretermino.

Se trata de femenino de 29 aos de edad, la cual acude
a urgencias debido a que desde hace 10 horas inicia
con contracciones dolorosas, la paciente cuenta con
antecedentes GO gesta 3, para 2, abortos 0, a la
exploracin fsica se observa producto ceflico, occipito
anterior derecho identificado por palpacin abdominal
y tacto vaginal, se observa a nivel de segundo plano, se
encuentra 100 % de borramiento y dilatacin de 7 cm,
la actividad uterina se observa 3 a 4 contracciones cada
10 minutos con duracin de 50 a 60 segundos, durante
la exploracin observa presencia de cordon umblical y
liquido amnitico de caractersticas adecuadas, el
producto presenta frecuencia cardiaca de 150 con
descensos variables durante la contraccin de menos
60 latidos.

PREGUNTA 378
Considerando el cuadro clnico, cual es la
conducta mas adecuada a seguir.

RESPUESTA
a.- Realizar Bloqueo y Realizar Cesarea.
b.- Realizar Bloqueo y Continuar Trabajo de Parto.
c.- Continuar Trabajo de Parto.
d.- Indicar Tocoliticos.

Se trata de femenino de 19 aos de edad la cual es
originaria del Estado de Mxico la cual acude a
urgencias, refiere que desde hace 6 horas presenta
dolor abdominal los cuales son espordicos,
aproximadamente cada 10 minutos, de intensidad
moderada, agrega que desde hace 4 horas arrojo moco
con sangre por va vaginal, durante la exploracin se
encuentra cuello cervical reblandecido, con un
centmetro de dilatacin con producto libre, usted
observa presencia de secrecin verdosa con olor
desagradable, la madre refiere que ha presentado
cuadro repetidos de infecciones cervicovaginales con
presencia de VPH por colposcopia reportado a las 18
semanas de gestacin, sin tratamiento, actualmente
cuenta con 40 semanas de gestacin.

PREGUNTA 379
Considerando el cuadro actual, cual es la
conducta a seguir:

RESPUESTA
a.- Esperar inicio de trabajo de parto.
b.- Tocolisis y antibioticoterapia.
c.- Prepara a la paciente para cesrea.
d.- Ingreso y perfil biofsico.

Acude a consulta paciente con 48 horas posparto,
refiere que presenta dolor en regin plvica de
caractersticas punzante e intensidad moderada a
severa, agrega que presenta cefalea continua, malestar
generalizado, fatiga, adinamia, cuenta con
antecedentes GO de gesta 3 para 3 abortos 0, refiere
que presento flujo amarillento en varias ocasiones en
este ultimo embarazo, a la exploracin se observa TA
100/50, FC 109 FR 34, Temperatura de 35.9 grados, a la
palpacin refiere dolor, al tacto se identifica liquido
serosanguinolento ftido y tero con flacidez, a las 3
horas de ingreso se presenta dificultad para respirar, a
la auscultacin se encuentra estertores bilaterales y
ataque al estado generalizado, TA de 70 /50 FC 142, FR
51, Temperatura de 36.1 grados.

PREGUNTA 380
Considerando la gravedad del cuadro cual es la
complicacin ms probable en esta paciente.

RESPUESTA
a.- Tromboembolia pulmonar.
b.- Coagulacion intravascular diseminada.
c.- Sepsis abdominal.
d.- Choque distributivo.

CASE REPORT
An 85-year-old white male with terminal pancreatic
cancer is expected to survive for another 2 weeks.
His pain has been satisfactorily controlled with
sustained-release morphine. He has now developed
a disturbed self-image, hopelessness, and
anhedonia, and has told family members that he has
thought about suicide. Psychomotor retardation is
also noted. His family is supportive. His daughter
feels he is depressed, while his son feels this is more
of a grieving process.
QUESTION 381
Which one of the following would be most
appropriate for managing this problem?

ANSWER
a) Reassurance
b) Methylphenidate
c) Trazodone
d) Alprazolam

QUESTION 382
A nurse who completed a hepatitis B vaccine series a
year ago is accidentally stuck by a needle that has
just been used on a dialysis patient. The patient is
known to be HBsAg-positive. Your first response
should be?

ANSWER
a) administer hepatitis B immune globulin (HBIG) only
b) provide reassurance only
c) repeat the hepatitis B vaccine series
d) test the nurse for hepatitis B antibody
QUESTION 383
A 30-year-old white primigravida asks you about the benefits
and drawbacks of corticosteroid therapy for premature labor
at 30 weeks gestation. Which one of the following statements
is most accurate?
ANSWER
a) Therapy is associated with a higher rate of persistent patent
ductus arteriosus
b) Weekly corticosteroid injections until 34 weeks gestation is
the standard regimen to prevent respiratory distress
syndrome
c) Therapy will decrease the risk of neonatal necrotizing
enterocolitis
d) Therapy is associated with a higher rate of neonatal
intraventricular hemorrhage

CASE REPORT

A nurse who completed a hepatitis B vaccine series a
year ago is accidentally stuck by a needle that has
just been used on a dialysis patient. The patient is
known to be HBsAg-positive.

QUESTION 384
Your first response should be?

ANSWER
a) administer hepatitis B immune globulin (HBIG)
only
b) provide reassurance only
c) repeat the hepatitis B vaccine series
d) test the nurse for hepatitis B antibody
CASE REPORT
A 62-year-old white male comes to your office with
pain and swelling of the left great toe at the
metatarsophalangeal joint. Examination shows it is
erythematous, warm, swollen, and tender to touch.
The patient has a history of diabetes mellitus
controlled by diet, and hypertension. , His
medications include hydrocWorothiazide, 25 fig/day.
A CBC and blood chemistry profIle are normal,
except for a uric acid level of 9.2 fig/dL (N 3.6-8.5).


QUESTION 385
Which one of the following is true in this situation?

ANSWER
a) The elevated uric acid level establishes the
diagnosis of gout
b) lntra-articular steroid injection should be avoided
c) Stopping the hydrochlorothiazide may control the
hyperuricemia
d) Allopurinol therapy should be started
QUESTION 386
Which one of the following is true regarding urinary tract
infections in febrile infants and young children (age 2-24
months)?

ANSWER
a) A perineal bag should be used to collect urine for
cultures
b) Prophylactic antibiotics should be continued for 1 year
in all children after the first febrile UTI
c) A 3-day course of oral antibiotic therapy is appropriate
d) Ultrasonography should be performed after the first
febrile UTI

QUESTION 387
Which one of the following intravenous agents given to
the mother during labor is the drug of choice for
prophylaxis of neonatal group B streptococcal
disease?

ANSWER
a) Erythromycin
b) Metronidazole
c) Clindamycin
d) Penicillin G

CASE REPORT
A 75-year-old Hispanic male presents with dyspnea on
exertion which has worsened over the last several
months. He denies chest pain and syncope, and was
fairly active until the shortness I of breath slowed
him down recently. You hear a grade 3/6 systolic
ejection murmur at the right ! upper sternal border
which radiates into the neck. Echocardiography
reveals aortic stenosis, with a mean transvalvular
gradient of 55 mm Hg and a calculated valve area of
0.6 cm2. Left ventricular function is normal.

QUESTION 388
Which one of the following is appropriate management
for this patient?

ANSWER
a) Aortic balloon valvotomy
b) Aortic valve replacement
c) Watchful waiting until the gradient is severe enough
for treatment
d) Medical management with B-blockers and nitrates

CASE REPORT
While performing a vasectomy on a 33-year-old male,
you are unable to definitely identify the left vas
deferens. You take some time to try and find the vas,
but the patient is becoming very uncomfortable. You
can easily isolate the right vas. The patient has
fathered three children and has no difficulty with
erectile function. The examination of his external
genitalia was normal.
QUESTION 389
What should you do next?

ANSWER
a) Abandon the attempt at vasectomy and
recommend a tubal ligation for his wife
b) Make a large scrotal incision and continue to try
to find the left vas deferens
c) Recommend that you try to find the left vas
deferens under general anesthesia
d) Ligate the right vas deferens and check a semen
specimen in 6 weeks.

QUESTION 390
A 27 year old white female sees you for the first time for a
routine evaluation. A Papanicolaou test reveals
atyrpical glandular cells of undetermined significance
(AGUS). Of the following, which one is most commonly
found in this situation?

ANSWER
a) Endometrial Hyperplasia
b) Endometrial Cancer
c) An Endocervical Polyp
d) Cervical Intraepithelial neoplasia
QUESTION 391
You perform a health maintenance examination on a 2-
year-old white male. He is asymptomatic and is meeting
all developmental milestones. The only significant
finding is a grade 3/6 diastolic murmur heard at the
right upper sternal border . Which one of the following
would be most appropriate at this time?

ANSWER
a) Referral to a pediatric cardiologist
b) Maintenance doses of digoxin
c) No further evaluation
d) Reevaluation in 6 months

QUESTION 392
A newborn term male infant at 2 hours of age is
noted by the nurse to be mildly jaundiced. Which
one of the following causes of jaundice can be
ruled out because of the infant' s age?

ANSWER
a) Concealed hemorrhage
b) Erythroblastosis fetalis
c) Congenital toxoplasmosis
d) Physiologic jaundice.
QUESTION 393
Which one of the following is indicated for treatment
of chlamydial urethritis during pregnancy?

ANSWER
a) Doxycycline
b) Ofloxacin
c) Ciprofloxacin
d) Erythromycin base
CASE REPORT
A 28-year-old female complains of generalized headache,
dizziness and generally not feeling well for 3 days. This started
at the same time as her menses and coincided with a major
examination in a college class she is taking. Her review of
symptoms is otherwise negative. Her past medical history
includes a recent acute onset of low back pain related to
lifting, and a recent depressive episode which responded well
to medication. Her current medications include an oral
contraceptive which she has taken for 2 years, a corticosteroid
nasal spray, and ibuprofen for the past 2 weeks. She was on
paroxetine, 30 mg/day, for 7 months, but this was stopped 5
days ago because of sexual dysfunction. Because of her
symptoms she has not taken any medications for the past 2
days. Since then the headache has eased substantially, but the
feeling of lightheadedness has remained
QUESTION 394
Which one of the following is the most likely cause of
her symptoms?

ANSWER
a) Paroxetine withdrawal
b) Stress
c) Serotonin syndrome
d) Viral infection

CASE REPORT
A 29-year-old gravida 0 para 0 presents to your office
with a chief complaint of irregular , infrequent
menses. Over the past few years, she has noted
increasing dark hair growth on her chin and above her
upper lip. On examination, she is normotensive and
moderately overweight. Examination of the skin
reveals acne and abdominal striae. The remainder of
the examination is normal. Laboratory Findings TSH
2.1JLU/mL(NO.3-5.0) FSH 8 U/L (N 1-10) LH 38 U/L(N
1-20) Free testosterone 50ng/dL(N 0.3-1.9) Prolactin.
28 ng/mL (N 0-23) 17 hydroxyprogesterone 4 ng/dL (N
0-8) DHEA -S ( dehydroepiandrosterone sulfate) 2.0
mg/dL (N 0-3.0)
QUESTION 395
In addition, a dexamethasone suppression test and
pelvic ultrasonography are normal. Which one of
the following is the most likely diagnosis for this
patient?

ANSWER
a) Cushing's syndrome
b) Virilizing adrenal tumor
c) Adult-onset congenital adrenal hyperplasia
d) Polycystic ovarian syndrome

QUESTION 396
Which one of the following patients should be screened
for hepatitis C virus (HCV) infection?

ANSWER
a) A 35-year-old nurse finishing her first year of work in
a dialysis unit
b) A 30-year-old day-care worker after a 1-year-old child
in her room was diagnosed with hepatitis C
c) A 45-year-old who received a blood transfusion in
1996 subsequent to a serious motor vehicle accident
d) A 9-month-old born to a mother positive for HCV

QUESTION 397
Which one of the following best describes the reason for
the marked decline in the incidence of epiglottitis in
the pediatric population?

ANSWER
a) More effective use of appropriate broad-spectrum
cephalosporin antibiotics
b) Increased use of the new polyvalent pneumococcal
vaccines
c) Immunization with Haemophilus influenzae type b
vaccine
d) Genetic drift in strains of parainfluenza virus

QUESTION 398
A 22 year old male has acute low back pain without
paresthesias or other neurologic signs. There is no
lower extremity weakness. Which treatment has
been shown to be of the most benefit initially?

ANSWER
a) Bed rest plus local injection of steroids
b) Complete bed rest for 2 weeks.
c) Low back strengthening program
d) Resumption of physical activity as tolerated

QUESTION 399
Which one of the following is a cause of
thyrotoxicosis characterized by a decreased
radioactive iodine uptake?

ANSWER
a) Subacute thyroiditis
b) Solitary toxic thyroid nodule
c) Graves' disease
d) Toxic multinodular goiter

CASE REPORT

A healthy 8-month-old white male has suddenly
developed recurrent bouts of what appears to be
severe abdominal pain. He cries inconsolably and
draws up his legs, but does not seem ill between
episodes. He has vomited and on examination has a
small amount of bloody mucoid stool in his diaper
and a palpable mass in his undistended right upper
abdomen.
QUESTION 400
The most likely diagnosis is?

ANSWER
a) pyloric stenosis
b) intussusception
c) gastroschisis
d) Meckel's diverticulum
QUESTION 401
A 17-year-o1d white female is brought in by her parents
because of concern about her recent weight loss. Her past
medical history is unremarkable. Physical examination is
notable for a body mass index of 23, bilateral parotid gland
enlargement, dental erosions on the lingual surfaces of her
teeth, submandibular lymphadenopathy, and elongated
abrasions on the dorsal surface of her right hand. The most
likely diagnosis is?

ANSWER
a) Sjogren' s syndrome
b) bulimia nervosa
c) sarcoidosis
d) HIV infection

QUESTION 402
A 10-year-old male is brought to your office after
sustaining a fall on an outstretched hand. Radiographs
show a nondisplaced fracture of the middle third of
the clavicle. Appropriate management would include
which one of the following?

ANSWER
a) Orthopedic consultation for surgical intervention
b) Weekly radiographs to assess callus formation
c) Intermittent heat to the area to help control pain
d) A figure-of-8 splint or sling support

QUESTION 403
A 10-year-old white male is brought to your office with a
chief complaint of "head congestion" associated with
moderate malaise and a low-grade fever for 7 days. He
has had a thick, discolored nasal discharge for the last 2
days. Which one of the following is correct regarding his
management?

ANSWER
a) Amoxicillin should be prescribed
b) Erythromycin should be prescribed
c) Sinus radiographs should be ordered, and the decision
to use antibiotics should be based on the findings
d) No antibiotics should be used at this time

QUESTION 404
A 19-year-old white male with a history of fever,
fatigue, weight loss, and mild diarrhea of 2 months'
duration is found to have a palpable mass in the right
lower quadrant of the abdomen. The most likely
diagnosis is?

ANSWER
a) lymphoma
b) Crohn's disease
c) amebic colitis
d) diverticulitis

Se trata de paciente masculino RN de 60 minutos,
obtenido por cesarea iterativa, dems de
presentar sufrimiento fetal, por disminucin de
actividad intrauterina y disminuciones de FCF, se
observa con movimientos muy finos, al inicio
presento cianosis leve y breve principalmente
periungeal y oral, se apresiaron secresiones
orales abundantes, a la exploracin fsica se
auscultan estertores bilaterales normales,
peristaltismo poco audible y temperatura de 36.1
grado, se apresi expulsin de secresiones con
tinte meconial durante la exploracin
PREGUNTA 405
Considerando el cuadro clnico cual es su
conducta a seguir?

RESPUESTA
a.- Paciente en observacin continua.
b.- Buscar posibles patologas ocultas.
c.- Realizar radiografia de torax.
d.- No hay datos patolgicos son adaptativo.

PREGUNTA 406
Cual es la conducta teraputica menos apropiada
en este momento del caso?

RESPUESTA
a.- Mantener ayuno hasta identificar causa.
b.- Colocar en ambiente neutro.
c.- Administrar oxigeno por casco ceflico 40 %.
d.- Gasometria y oximetra de pulso.

PREGUNTA 407
Cual de los siguientes parmetros gasomtrico no
es normal esperarlo en el caso?

RESPUESTA
a.- pH < 7.34.
b.- PaCO > 45 mmHg.
c.- HCO3 > 40mEq/L.
d.- SpO2 < 95.

PREGUNTA 408
Se realizaron electrolitos sericos por continuar con
rechazo al alimento y secresion moderada oral, todos
los resultados resultaron dentro de parmetro
normales, cual de las siguientes diagnosticos
diferenciales del acidosis respiratoria es el mas
probable presentarse?

RESPUESTA
a.- Ventilacin inadecuada alveolar.
b.- Desordenes musculares.
c.- Defectos pulmonares.
d.- Trastornos de las vas areas.

PREGUNTA 409
Considerando los valores gasomtricos y de
eletrolitos, cual es la causa de la modificacin del
anio gap, del caso?

RESPUESTA
a.- Error de laboratorio.
b.- Toxinas no identificadas.
c.- Mielomas.
d.- Exceso de soluciones.

PREGUNTA 410
El paciente presenta persistencia de los sntomas
por 3 horas ms, se reporta que incrementa con
la alimentacin, con dificultad continua
succionando, usted considera realizar
laboratorios, cual de los siguientes resultados es
el menos probable encontrar en este caso?

RESPUESTA
a.- Aumento de la trama vascular.
b.- Presencia de infiltrados pulmonares.
c.- Acidosis respiratoria.
d.- Hipoxemia leve.

Se trata de paciente masculino de 5 horas de vida
estrauterina, el paciente naci via cesarea, naci
a la 35 semanas de gestacion, se administro
maduradores a la madre debido a trabajo de
parto distcico con ruptura de membranas 32
horas antes de la intervencin quirrgica, a la
exploracin fsica se observo taquipnea de 101
rpm, y signo moderados de dificultad
respiratoria, resto de parmetros maternor y del
neonato dentro de limites normales.

PREGUNTA 415
Cual de los diagnsticos diferenciales es menos
probable encontrar?

RESPUESTA
a.- Sndrome de adaptacin pulmonar.
b.- Sndrome de aspiracin de meconio.
c.- Enfermedad de membrana hialina.
d.- Neumona neonatal hospitalaria.

PREGUNTA 416
Se decide realizar radiografia de torax, cuales son
los datos no es probable espera encontrar para
confirmar el diagnostico?

RESPUESTA
a.- Broncograma areo.
b.- Hilio congestivos.
c.- Derrame cisurales.
d.- Sobredistencion pulmonar.

PREGUNTA 417
Cual es el fenmeno fisiopatolgico no es la mas
adecuada para el caso?

RESPUESTA
a.- Falta de absorcin de lquido amniotico.
b.- Falta administracin de esteroides.
c.- Deficienca ligera de surfactante.
d.- Edema pulmonar transitorio.

Un RN de 3,6 kg naci a las 37 semanas de
gestacin hijo de diabtica. Obetnido por cesrea
con Apgar de 6/9. Desarrollo taquipnea
inmediatamente despus de su nacimiento y
requirio oxgeno suplementario. En la gasometria
presento; fueron PO2 de 57 mm Hg, PCO2 de 52
mm Hg, y pH de 7,31. El nio se mantuvo en
oxgeno por campana. A las 2 horas el paciente se
encuentra hipotnico, ciantico, con saturacin
del 70 %. La Rx de observa rectificacin,
hiperclaridad e incremento de los espacios
intercostales y congestin parahiliar.

PREGUNTA 418
Cual es la medida inmediata a seguir.

RESPUESTA
a.- Intubacion orotraqueal.
b.- Alimentacin por sonda orogastrica.
c.- Mantener un ambiente neutro.
d.- Realizar medidas de reanimacin.

PREGUNTA 419
Cual de las siguientes datos radiolgicos es
menos probable para el diagnostico?

RESPUESTA
a.- Aumento de la trama vascular con imgenes
algodonosas.
b.- Liquido en cisusras interlobales.
c.- Abatimiento de diafragma.
d.- Cardiomegalia.

RN masculino de edad gestacional de 39
semanas, de 3,8 kg hijo de madre diabtica el
cual se obtuvo por cesaria previa administracin
de esterioides a la madre, el cual requiri de
estimulacin vigorosa cursando con hipotermia
que mejoro en incubadora, se observo
posteriormente, aleteo nasal, retraccin esternal,
quejido respiratorio, cianosis y polipnea, la Rx
mostro un patro retculo granular difuso bilateral
y disminucin de la expansin pulmonar, PaO2 <
50 mmHg.

PREGUNTA 420
Cual es la conducta a seguir.

RESPUESTA
a.- Presin positivo nasal continua.
b.- Oximetria de pulso.
c.- Intubacin traqueal.
d.- Ventilacin mecnica.

RN femenino de trmino obtenido por parto vaginal
espontneo con un peso al nacer de 3.500 g. el
monitoreo intraparto no revelaron evidencia de
sufrimiento fetal. Poco despus del parto, fue
ingresado en una unidad de cuidados intensivos debido
a la presencia de meconio, vmitos y succin debil, su
temperatura era inferior a 36,0 C, el pulso fue de 148
lat / min, y su frecuencia respiratoria era de 72
respiraciones / min. El tiempo de protrombina y el
tiempo parcial de tromboplastina activada eran 20,7 y
54,6 s, respectivamente. La orina y sangre se
sometieron a la cultivo. La radiografa de trax mostr
infiltrados y rayas gruesas del campo pulmonar
derecho.

PREGUNTA 421
Considerando la gravedad del cuadro cual es la
complicacin ms probable en esta paciente.

RESPUESTA
a.- Sepsis.
b.- Acidosis respiratoria.
c.- Neumonia.
d.- Coagulopatia.

PREGUNTA 422
Cual de las siguientes manifestaciones es mas
importante para el pronostico?

RESPUESTA
a.- La cantidad de meconio aspirado.
b.- La edad gestacional.
c.- Manifestaciones de neumonitis qumica.
d.- Manifestaciones de posmadurez.

PREGUNTA 423
Cual de las siguientes alteraciones es importante
mantener en la estabilizacin mediata del
paciente?

RESPUESTA
a.- Taquipnea.
b.- Cianosis persistentes
c.- Desaturaciones.
d.- Cambios minimos se inestabiliza.

PREGUNTA 424
Cual de las siguientes medidas es menos
importantes en la reanimacin del paciente?

RESPUESTA
a.- No estimular.
b.- No frotarlo.
c.- No voltearlo ni manipular.
d.- Aspirar primero la nariz y la orofaringe,

PREGUNTA 425
Cual de las siguientes manifestaciones es mas
frecuente encontrar en esta patologia?

RESPUESTA
a.- Infiltrados.
b.- Hiperinsuflacion.
c.- Atelectasia semegtadas o globales
d.- Puede haber neumotrax.

Paciente masculino 3 dias que presenta datos de
dificultad respiratoria con rechazo a la
alimentacin, su nacimiento fue a las 35 semanas
de gestacion, cabe destacar presencia de
conjuntivitis, parto distocico, por via vaginal, la
radiografia es inespecficas, solo un infiltrado
denso alveolar, la imagen fue persistente, los
laboratorios mostraron eosinofilia, no recibi
factor surfatante pero recibi oxigeno al 100 %,
cabe destacar que mientras se encontraba en
unidad de cuidados intensivos presento
distencin abdominal.

PREGUNTA 426
Cual es la conducta farmacologa a seguir considerando
que los cultivos aun no tienen datos para ser dirigido?

RESPUESTA
a.- Vancomicina 74mg/Kg y cefotaxima 50mg/kg
b.- Cefotaxima 20 mg/ Kg y metronidazol 30 mg/Kg
c.- Ampicilina 100 mg /kg mas amikacina 15mg/kg
d.- Doxiciclina 15 mg/kg mas gentamicina 15 mg/kg

Paciente masculino 10 dias de nacimiento, que se
obtuvo en casa sin control prenatal, ingresa por
diarrea, llanto, e irritabilidad, vomito en proyectil,
en casa, refiere la madre que solo fue en una
ocacion, al ingreso se observo hipotnico, llanto
agudo, mal estado generalizado, con tono cervical
no se observo rigidez, pero fontanela abombada,
su peso fue de 2050 grs, laboratorios con 150,000
de plaquetas, antecedentes de rinorrea hialina,
se presento 35 grados, irritabilidad, durante la
exploracin se observo pedaleo y chupeteo,
dificultad respiratoria.

PREGUNTA 427
Para establecer el diagnostico y conducir su
teraputica para el caso?

RESPUESTA
a.- Hemocultivo.
b.- Punsion lumbar.
c.- Tomografia.
d.- Biometria hemtica.

PREGUNTA 428
Se encuentra en espera de resultados, sin embargo el
paciente se va deteriorando, cual es la agente
etiologico mas probable para dar una teraputica
dirigida?

RESPUESTA
a.- H influenza
b.- E Coli.
c.- Listeria monocitogenes
d.- Estreptoco b-hemolito.

Se trata de paciente masculino de 37 semanas de
gestacion obtenido por cesarea, la madre presento
cuadro de pre-eclampsia, luego de 24 horas de nacido
presenta disminucin de la succion, letargia, falta de
respuesta a estimulos con disminucin de tono
muscular, asi como alteraciones de la temperatura con
tendencia a la hipotermia, los laboratorios reportaron
35 mg/dl de glucosa perifrica, resto de datos de
laboratorio y gabinete dentro de parmetro normales,
sin embargo posterior a la administracin de glucosa
mediante catter perifrico presenta nuevos cuadro de
hipoglucemia?
PREGUNTA 429
Cual de los siguiente antecedentes es menos
frecuente para la patologia que presenta el
paciente?

RESPUESTA
a.- Displasia de clulas beta.
b.- Enfermedad de orina de arce.
c.- Galactosemia.
d.- Hipotiroidismo congnito.

Al segundo da de vida presenta resolucin de su dificultad
respiratoria con disminucin de requerimientos de oxgeno
y tolerancia al destete, por lo cual se inicia estmulo enteral
con adecuada tolerancia. Los laboratorios iniciales
hemograma normal y PCR negativa. Se contino vigilancia
clnica y aumento progresivo de la va oral. A los 5 das de
vida presenta cuadro de episodio emtico de contenido
alimentario sin otra sintomatologa el cual se interpret
como reflujo gastroesofgico. Al da siguiente presenta
deterioro clnico dado por taquicardia, distensin
abdominal, persistencia de episodios emticos
postprandiales, residuo gstrico del 70%, deposiciones con
sangre macroscpica e hipoglicemia.

PREGUNTA 430
Cual es la conducta a segur mas adecuada en este
momento?

RESPUESTA
a.- Se suspende la va oral, BH, QS, cultivos, ampicilina y
gentamicina.
b.- Ampicilina + amikacina, Bh, QS, cultivos, ayuno.
c.- Alimentacion parenteral, antibioticoterapia empirica,
BH, QS, cultivos.
d.- Ampicilina, cefotaxima, alimentacin por sonda, BH y
QS.

Un varn de 11 meses alimentado
exclusivamente a seno materno, vive en zona
rural y es el ultimo hijo de 8 en total, madre
dedicada al hogar con educacin primaria,
reingres 10 das despus de un episodio previo
de gastroenteritis con diarrea durante 2 das y
deshidratacin, Despus de la rehidratacin
estaba por debajo del 3er percentil para el peso.
2 semanas despus de la admisin la diarrea es
severa. Una semana ms tarde la diarrea era con
sangre, fiebre. Egresa una semana despus de
tratamiento intrahospitalario.

PREGUNTA 431
El paciente anterior reingresa por presentar irritabilidad,
llanto sin lagrimas e incontrolable, hipotnico, hipotrmico
con lienzo hmedo y reporta hipoglucemia la cual es
controlada oralmente. Considerando la comorbilidad de la
desnutricin cual es la conducta profilctica a seguir.

RESPUESTA
a.- Administrar trimetropin sulfametoxazol.
b.- Administrar cefotaxima.
c.- Administrar ampicilina ms gentamicina.
d.- Administrar cloranfinicol IV.

Un nio de 12 aos de edad se present con lesiones
generalizadas en la piel eritematosas y costras de 4 meses de
duracin. Las lesiones aparecieron por primera vez en su cuero
cabelludo y dentro de unos das, eritrodrmica se generaliz y, a
continuacin. Escalado y exudacin tambin fueron vistos.
DiferenteS antibiticos y esteroides tpicos fueron prescritos sin
mejora significativa. El nio tambin se quejo de prdida de la
audicin desde un mes. En el examen fsico, eritrodermia con
escalamiento grave y maloliente con descarga y leve
queratodermia palmoplantar todo el cuero cabelludo. Haba dos
pequeas vesculas a lo largo del lado cubital de la palma
derecha. Las superficies de las mucosas y las uas eran
normales. Tena ganglios linfticos submandibulares. El conducto
auditivo estaba lleno de escamas y costras. pabelln de la oreja
era sensible a la palpacin. Despus de quitar las costras, canal
auditivo externo se encontr que era roja e hinchada

PREGUNTA 432
Cual es la conducta a seguir.

RESPUESTA
a.- Prednisona oral de 0.5 a 1 mg/kg al da.
b.- Dapsona dosis de 100 mg/da.
c.- Azatioprina es de 2.0 a 2.5 mg/kg,
d.- Ciclofosfamida a dosis oral de 1 a 3 mg/kg al
da

Nio de 6 aos de edad, sin antecedentes
personales ni familiares de inters, que fue
valorado en el servicio de urgencias por
presentar, desde haca 24 h, lesiones cutneas
maculopapulosas y purpricas distribuidas
simtricamente por las nalgas, miembros
inferiores y superiores asociadas a signos
inflamatorios en las rodillas y tobillos. Los anlisis
de orina, de funcin renal y la presin arterial
eran normales. Se inici tratamiento con
antiinflamatorios no esteroides y se dio de alta.

PREGUNTA 433
Cual de los siguientes factores etiolgicos puede estar
ms relacionado con esta patologia.
RESPUESTA
a. Estreptococo del grupo A beta hemoltico o M.
pneumoniae,
b.- Virus de hepatitis A, citomegalovirus virus de
Ebstein Barr.
c.- Vacuna del sarampin, influenza, rubola,
neumococo.
d.- Antibiticos betalactmicos, macrlidos AINES.

Nio de 10 aos que consult al servicio de
urgencias despus de presentar crisis convulsiva
tonicoclnica generalizada (TCG) de 3 min de
duracin, despus de 4 das de fiebre, tos,
inapetencia general y fatiga; en el hospital curs
febril, con debilidad, desorientado; unos minutos
despus present otra crisis TCG de 3 min, se
controla las crisis con mejora 12 horas despus
presenta las siguientes manifestaciones
Inflitrados pulmonares en ms de dos cuadrantes,
ndice de oxigenacin PaO2/FiO2 menor de 250 y
distensibilidad pulmonar disminuida.

PREGUNTA 434
Cual es la conducta a seguir.

RESPUESTA
a.- Oseltamivir 60 mg cada 12 hrs por 5 das.
b.- Ceftriaxona 50-100 mg/kg/da IV c/24 hrs por 7-10
das.
c.- FiO2: el necesario para mantener PaO2 arriba de 60
mm Hg.
d.- Presin menor de 35 cm H2O y meseta menor de
30 cm H2O.

Ingresa masculino de 4 aos de edad el cual inicia
padecimiento sbito con estridor larngeo
inspiratorio, refiere la madre que hace dos das
inicio con tos, rinorrea y fiebre no cuantificada,
motivo por el cual acudi a consulta en centro de
salud donde fue indicado tratamiento con
analgsico, medidas generales y datos de alarma,
agrega que una hora antes del ingreso se
incrementaron los sntomas con dificultad para
hablar, inquietud, llanto con disfona y tos seca.

PREGUNTA 435
Cual es la conducta teraputica a seguir.

RESPUESTA
a.- Administracion de dexametasona .60mg/Kg.
b.- Budesonida inhalada.
C.- Fluquitasona.
d.- L-epinefrina nebulizada 1:1000

Masculino de 3 aos de edad con sndrome de Down y
cardiopata congnita, es trado a urgencias por cuadro
catarral de 7 das de evolucin con tratamiento
sintomtico el cual mejoro parcialmente pero fue
lentamente reagudizado, durante las dos semanas
previas cambio en dos ocaciones de tratamiento, existe
el antecedente que sus dos hermanos presentaban
cuadro catarral. A las 12 h del ingreso presenta
empeoramiento progresivo de la dificultad respiratoria
con aparicin de fiebre, secreciones mucopurulentas y
tos continua, taquicardico y taquipnea.

PREGUNTA 436
Cual es el agente etiolgico mas probable de este
caso.

RESPUESTA
a.- Influenza A, influenza B, parainfluenza.
b.- Virus sincitial respiratorio.
c.- Coronavirus, adenovirus y rinovirus.
d.- Bordetella pertusis, Mycoplasma pneumoniae
y C pneumoniae.

Un nio de 5 aos de edad se present con fiebre y tos no
productiva durante 3 das. El examen fsico revel mltiples
ndulos linfticos cervicales. Fue tratado en su centro de
salud durante 7 dias con medidas generales, regresando a
su escuela sin embargo a los 5 dias despus inicia con tos
productiva, dificultad respiratoria, cansancio, malestar
generalizado, aleteo nasal y fiebre de 39.2 grados. Examen
torcico se apresiaron estertores bronquiales y
broncoalveolares de predominio en la zona inferior. La
saturacin de oxgeno de 92% en aire ambiente. La
radiografa de trax (RXT) al ingreso mostr consolidacin
del lbulo inferior derecho. Leucositos de 13,500. Refiere el
familiar que el esquema de vacunacin se encuentra al
corriente. Vive en zona rural y es el segundo hijo de 5.

PREGUNTA 437
Cual es la conducta terapeutica mas apropiada.

RESPUESTA
a.- Amoxicilina 80-90 mg/kg/dia.
b.- Azitromicina 10 mg/kg dia.
c.- Eritromicina 50mg/Kg/dia.
d.- Penicilina procainica 400,000 UI cada 24 hrs
IM.
Nia de 7 aos que acude a urgencias por cuadro de
tos, dificultad respiratoria de dos semanas de
evolucin y pico febril 38C. con antecedentes
patolgicos de obesidad, roncadora habitual y
respiracin bucal. adenoidectomizada hace 2 aos por
sospecha clnica de SAHS sin control posterior. Peso
56kg (p>97), FC 135 lpm (p>95), TA 109/70, SatO2 82%
con FiO2 21%. Facies anmica, implantacin baja del
pelo, estrabismo y obesidad mrbida. Aceptable
estado general, normohidratada, palidez cutnea y
retracciones subcostales moderadas. Auscultacin
cardiaca normal. Auscultacin respiratoria con
hipoventilacin generalizada moderada, sibilantes
espiratorios finos y roncus dispersos.

PREGUNTA 438
Cul es la conducta farmacolgica mas adecuada.

RESPUESTA
a.- Salbutamol, bromuro de ipatropio inhalados y
corticoides endovenosos.
b.- Salbutamol, corticoide, loratadina.
c.- Salbutamol, oxigeno, ambroxol y prednisona.
d.- Ambroxol, prednisona, Bromuro de ipatropio y
oxigeno
Femenino de 3 aos de edad la cual inicia 3 dias
previos a su ingreso, la madre refiere diarrea con
moco y sangre, asi como fiebre alta no
cuantificada, esto se acompao de rinorrea y
dificultad para consiliar el sueo, a la exploracin
fsica se observa, decada con aspecto
deteriorado y respuesta alterada irritable, signos
vitales taquicardico y taquipnea, se observa
disminucin de la turgencia de la piel, se ingresa
a urgencias para tratamiento.

PREGUNTA 439
Cual es el volumen recomendado de SRO para
este caso.

RESPUESTA
a.- SRO 30 ml/kg durante 4 hrs.
b.- SRO 50 ml/kg la 1era hora luego 30 ml/Kg.
c.- SRO mayor a 50 ml/kg durante 4 horas.
d.- Plan de hidratacion intravenoso.

Una nia de 4 aos de edad con dolor abdominal y
distensin desde hace 6 das, segn su madre. Ella tambin
tena antecedentes de vmitos desde 3 das. El dolor de
inicio gradual, clicos, seguido por distensin abdominal y
aumento en la intensidad del dolor y los vmitos. El vmito
era de color verdoso y contena partculas de alimentos no
digeridos. El nio fue vacunado parcialmente. En la
exploracin fsica estaba mal alimentado con palidez,
temperatura oral fue de 37,7 C, la presin arterial era de
98/60 mmHg, pulso regular, con una frecuencia de 110
latidos por minuto, frecuencia respiratoria de 34 por
minuto. El examen abdominal revel un abdomen
distendido de todos los cuadrantes. En la auscultacin,
ruidos intestinales hiperdinmico eran audibles. Examen
rectal revel heces con sangre y moco asi como varios
gusanos vivos.

PREGUNTA 440
Considerando el agente etiolgico mas probable en el caso
anterior, cual es la conducta teraputica a seguir.

RESPUESTA.
a.- Iodoquinol 30-40 mg/kg/da vo (mx 2 g) en 3 dosis x 20
da, albendazol 15 mg/kg/da vo x 5-7 das (max 400 mg).
b.- Metronidazol 5 mg/kg vo tid (mx 750 mg/da) x 5 das,
nitazoxanida 200 mg x 3 das.
c.- Tetraciclina 40 mg/kg/da (mx 2 g) vo en 4 dosis x 10
das, metronidazol 20-40 mg/kg/da vo en 3 dosis x 10 das.
d.- Albendazol 15 mg/kg/da vo x 5-7 das (max 400 mg),
metronidazol 5 mg/kg vo tid (mx 750 mg/da) x 5 das.

Una lactante de 17 meses de edad, present
irritabilidad aguda, llanto, dolor en el pie izquierdo. La
madre de la paciente le quit los zapatos y se encontr
una viuda negra muerta en el zapato izquierdo del
nio. Se desarrollo rpidamente inflamacin del pie
izquierdo. El EF mostro presin arterial 145/103 mm Hg
y taquicardia (frecuencia cardaca: 160-180 latidos por
minuto). El examen revel edema y eritema en el pie
izquierdo y los prpados, y ninguna lesin diana clsica
vista en el sitio. Se observaba con dolor significativo. La
glucosa srica fue elevada a 186 mg / dl, y su recuento
de glbulos blancos se aument a 17 000/mm3.

PREGUNTA 441
Cual es el pronstico de la aracnismos presuntivo del caso.

RESPUESTA
a.- Dolor agudo, opresin precordial, taquicardia, dificultad
respiratoria y sensacin de muerte inminente.
b.- Alteraciones electrocardiogrficas, hipertensin y
edema de pulmn.
c.- Sensacin punzante, dolor urente o quemante y edema
local.
d.- Ppula, que puede evolucionar a una placa eritematosa
o placa marmrea.

Se trata de una nia de once aos que presentaba:
boca seca, confusin, habla incoherente, incapacidad
para reconocer a los miembros de la familia, que
tambin se presenta vmito incontrolable, trastornos
visuales, auditivos y alucinaciones visuales. El examen
clnico revel alteracin de la conciencia, coma la
escala de Glasgow (GCS) se evalu a 13/15, las pupilas
eran iguales y reactivas, present polipnea a 26 ciclos
por minuto, pero estaba afebril y hemodinmicamente
estable, con ictericia mucocutnea, reflejos tendinosos
eran agudos y difusos. Recuento de sangre de rutina
completo, pruebas de funcin renal y heptica revel
citolisis heptica.
PREGUNTA 442
Cual es la conducta a seguir.

RESPUESTA
a.- Realizar lavado gstrico.
b.- Administrar diazepam 5 mg.
c.- Administrar anticolinergico.
d.- Administrar fisostigmina.
Varn de 15 aos, ingres tras sufrir un episodio de
parestesias en el hemicuerpo izquierdo, junto con
desviacin de la comisura bucal hacia la izquierda, seguido
de cefalea frontal intensa y pulstil, nuseas y vmitos.
Antecedentes: producto de un embarazo y parto normales
con un desarrollo psicomotor y del lenguaje adecuados,
con trastorno del aprendizaje. No present antecedentes
familiares de inters excepto que su abuelo (materno)
sufra de migraas. A los 4 aos, tras un traumatismo
craneoenceflico leve, sufri un cuadro de inestabilidad y
somnolencia de varias horas de duracin. A los 5 aos
present en vigilia, de forma sbita, una hemiparesia aguda
izquierda asociada a disartria y confusin, que cedi en 10
minutos. El examen fsico y neurolgico completo
evidenci, una leve dismetra del miembro superior
izquierdo.

PREGUNTA 443
Cul de los siguientes diagnosticos es el ms
probable.

RESPUESTA
a.- Crisis parciales simples.
b.- Crisis parciales complejas.
c.- Crisis generalizadas convulsivas.
d.- Crisis generalizadas no convulsivas.

PREGUNTA 444
Cual es el auxiliar diagnostico mas adecuado para
identificar la causa.

RESULTADO
a.- MVEEG.
b.- EEG.
c.- IRM.
d.- TAC.

PREGUNTA 445
Cual es la conducta teraputica mas adecuada
para el caso.

RESPUESTA
a.- Carbamazepina 10 a 20 mg/kg.
b.- Lamotrigina 200 a 400 mg/da administrados
en dos dosis.
c.- Valproato de magnesio 60 mg/kg/da.
d.- Topiramato 100 mg/dia.

CASE REPORT

A 78-year-old white female presents with a 3-day history of
lower thoracic back pain. She denies any antecedent fall or
trauma, and first noted the pain upon arising. Her
description of the pain indicates that it is severe, bilateral,
and without radiation to the arms or legs. Her past medical
history is positive for hypertension and controlled diabetes
mellitus. Her medications include hydrochlorothiazide,
enalapril, metformin, and a general multivitamin. She is a
previous smoker but does not drink alcohol. She underwent
menopause at age 50 and took estrogen for a few months
for hot flashes. Physical examination reveals her to be in
moderate pain with a somewhat stooped posture and mild
tenderness over T12L1. She has negative straight-leg
raising and normal lower extremity sensation, strength, and
reflexes.
QUESTION 446
Which one of the following is true regarding this patients
likely condition?

ANSWER
a) Subcutaneous or intranasal calcitonin may be very
helpful for pain relief
b) Investigation for an underlying malignancy is
indicated
c) Prolonged (approximately 2 weeks) bed rest will
increase the chance of complete
recovery
d) An MRI or nuclear medicine bone scan should be
performed

QUESTION 447
A 5-year-old male is scheduled for elective hernia
repair at 11:00 a.m. Which one of the following
would be the most appropriate recommendation?

ANSWER
a) No solid food for 8 hours prior to surgery and clear
liquids until 2 hours prior to surgery
b) No solid food after midnight and nothing by mouth
8 hours prior to surgery
c) Nothing by mouth 8 hours prior to surgery
d) Nothing by mouth 2 hours prior to surgery

CASE REPORT
A 4-year-old male has a fever of 1 weeks duration. It
has been at or slightly above 38 C (101F) and has
responded poorly to antipyretics. The patient
complains of photophobia, burning in his eyes, and a
sore throat. His mother also notes that his eyes look
red, his lips are red and cracked, and he has a
strawberry tongue. The childs palms and soles are
erythematous and the periungual regions show
desquamation of the skin. He has minimally painful
nodes located in the anterior cervical region, about
22 cm in size. A Streptococcus screen is negative.
QUESTION 448
The most appropriate management at this time
would be?

ANSWER
a) intravenous nafcillin
b) intramuscular benzathine penicillin G 600,000 U
c) intravenous immune globulin and aspirin
d) prednisone, 23 mg/kg daily
CASE REPORT
A 6-year-old white male visits your office with chief
complaints of a recent onset of fever, bilateral knee
and ankle pain, colicky abdominal pain, and rash. On
examination, his temperature is 38.3 C (101.0 F),
and there is a prominent palpable reddish-brown
rash on the buttocks and thighs. There is pain on
motion of his knees and ankles, and mild diffuse
abdominal tenderness. The stool is positive for occult
blood. Laboratory Findings: Hemoglobin 11.0 g/dL
Hematocrit 33% WBCs 14,500/mm; 85% segs, 3 15%
lymphs, Platelets 345,000/mm 3 Prothrombin time
12 sec
QUESTION 449
Which one of the following is the most likely diagnosis?

ANSWER
a) Henoch-Schnlein purpura
b) Rocky Mountain spotted fever
c) Acute iron ingestion
d) Systemic onset juvenile rheumatoid arthritis

CASE REPORT

A 76-year-old white male with a history of recurrent
depression has recently become more depressed and
developed psychotic features. His symptoms have
not responded to antidepressants and antipsychotic
agents, prescribed by his psychiatrist. The
psychiatrist has recommended electroconvulsive
therapy (ECT) for the patient. The patients family
visits you to ask for your opinion and
recommendations regarding ECT in this individual.
QUESTION 450
In your consultation with this family, which one of the
following would be accurate advice regarding ECT?

ANSWER
a) It has a low response rate
b) It is efficacious and safe
c) There is evidence that it predisposes to the
development of dementia
d) It causes irreversible short-term memory loss


CUARTO SIMULADOR DEL CURSO
ENARM CMN SIGLO XXI

Das könnte Ihnen auch gefallen